Which of the following examples best illustrates the concept of approach-approach conflict?

1. Jill wants to study the process of thinking. Which field of psychology should she choose? (A) Cognitive (B) Social (C) Personality (D) Learning (E) Perception

1. (A) Cognitive psychology is the study of how we process, store, and retrieve informa- tion. Choices (B) and (C) are devoted to studying the way people relate to others and the unique attributes of a person; neither field focuses on one's thought process. (D) deals with long-lasting changes in behavior, usually through experience. (E) is the experience of a meaningful pattern of a stimulus.

2. I believe people choose to live meaningful lives. I share many of the same beliefs as Carl Rogers. Most important, I believe many people have the ability to reach self-actualization. Who am I? (A) Wertheimer (B) Skinner (C) Maslow (D) Terman (E) Seligman

2. (C) Abraham Maslow is a humanist. The humanist approach emphasizes that each indi- vidual has free will to determine his or her own future. Self-actualization is an inherent tendency to reach our true potential.

3. Of the following, who is associated with the Gestalt school of psychology? (A) John Watson (B) William James (C) Ivan Pavlov (D) Max Wertheimer (E) Sigmund Freud

3. (D) Wertheimer, along with Wolfgang Kohler and Kurt Koffka, studied the illusion of flashing lights and the perception of movement. Wertheimer argued that perceptual expe- riences, such as flashing lights, resulted from a "whole pattern" or, in German, "Gestalt."

4. Which of the following psychologists wrote The Principles of Psychology? (A) William James (B) Wilhelm Wundt (C) John Watson (D) Sigmund Freud (E) Max Wertheimer

4. (A) William James wrote the Principles of Psychology, published in 1890. This book included the study of the mind, sensation, memory, and reasoning. James is associated with functionalism. Wundt is associated with structuralism. Watson is associated with behavior- ism. Freud is associated with psychoanalysis. Wertheimer is associated with Gestalt.

5. Psychology is considered a science mainly because it relies on direct observation. Which field of psychology supports this? (A) Behaviorism (B) Psychodynamic psychology (C) Social psychology (D) Cognitive psychology (E) Structuralism

5. (A) John Watson published a paper called "Psychology as a Behaviorist Views It." Wat- son rejected the notion that introspection can be used as a technique to determine the behavior of human beings. Watson believed psychology needed to be an objective experi- mental science. Unlike choices (B), (C), (D), and (E), behaviorism was the first field to study psychology in an observable and measurable manner.

6. Which of the following best defines eclectic psychology? (A) The study of animal instinct (B) The study of child development (C) The study of abnormal behavior (D) The study of a variety of theories with in the field (E) The study of the human brain and central nervous system

6. (D) The definition of the eclectic approach is a combination of techniques and ideas from many different schools of thought in psychology.

7. Psychoanalytic psychology focuses mainly on: (A) Rewards and punishments (B) Self-esteem and self-actualization (C) Biology and genetics (D) Internalconflictandunconsciousdesires (E) Sensation and perception

7. (D) The psychoanalytic approach focuses on the idea that each of us has an unconscious that contains thoughts, desires, and fears that have been hidden or repressed because they threaten our conscious self. (A), rewards and punishments, is based on behaviorism. (B), self-esteem and self-actualization, is based on humanism.

8. One major criticism of Ivan Pavlov's concept of classical conditioning was that: (A) It did not take into account voluntary human behavior. (B) It was unethical to use dogs in a psychology experiment. (C) It did not take into account involuntary behavior. (D) The findings over lapped with other fields of psychology. (E) It did not relate to human behavior.

8. (A) In Pavlov's experiment in which he rang a bell before putting food in the dogs' mouths, the dogs eventually paired the bell with salivating, even when the food was not present. This phenomenon, which Pavlov called conditioned reflex, eventually became known as classical conditioning. Because this theory was based on involuntary reflexes and many psychologists believe human behavior is based on voluntary choices, they criticized classical conditioning, claiming it could not help us further understand human behavior. This explanation negates choice (C). (D) and (E) are irrelevant to this question.

9. Which of the following psychologists was a structuralist? (A) John Watson (B) Wilhelm Wundt (C) William James (D) MaxWertheimer (E) Sigmund Freud

9. (B) Wilhelm Wundt established the first psychological laboratory in 1879. Structuralism is the study of the most basic elements in our conscious minds. John Watson was a behaviorist. William James studied functionalism. Max Wertheimer studied Gestalt. Sigmund Freud studied psychoanalysis.

10. The use of rewards, punishments, and positive reinforcement is an example of which field of psychology? (A) Personality (B) Behavioral (C) Social (D) Cognitive (E) Psychoanalytic

10. (B) The behavioral approach analyzes how organisms learn or modify behavior based on rewards and punishments in the environment. The other choices do not specifically focus on reinforcements in one's environment.

11. "Give me a dozen healthy infants and my own special world to bring them up in, and I'll guarantee to take any one at random and train him to become any type of specialist, . . . lawyer, doctor . . ." What psychological approach would support this statement? (A) Cognitive (B) Structural (C) Functional (D) Behavioral (E) Psychoanalytic

11. (D) The behavioral approach emphasizes the objective, scientific analysis of observ- able behavior. This includes conditioning human behavior. Choice (A) focuses on an indi- vidual's thought process or perception. Choices (B) and (C) were both schools of thought that focused on introspection. Psychoanalysis emphasized the strength of the unconscious.

12. Who was considered the father of psychology? (A) James (B) Wundt (C) Wertheimer (D) Freud (E) Kohler

12. (B) Wilhelm Wundt is considered the father of psychology. Wundt established the fi rst psychological laboratory in 1879.

13. One major difference between structuralism and functionalism is: (A) Structuralists analyze all mental elements, while functionalists analyze only some elements. (B) Structuralists believe all behaviors stem from the evolutionary process. (C) Structuralists wish to divide the mind into mental elements while functionalists believe behavior helps an organism adapt to the environment. (D)Only functionalists believe in the importance of introspection. (E) Structuralists try to manipulate the mind in order to understand behavior, while functionalists study the conscious mind to understand behavior.

13. (C) Structuralism was infl uenced by the physical scientists of the time. Wundt emphasized that all complex substances could be separated into component elements, whereas functionalists examined behaviors from a diff erent point of view. Functionalists were asking what the mind does and why. Choice (C) best exemplifi es these concepts. (A) is too vague and inaccurate to be the correct answer. (B) does not represent either structuralism or functionalism. (D) is incorrect because both structuralism and functionalism used introspection as a means of determining human behavior. Once again, choice (E) is not using accurate information to defi ne either structuralism or functionalism

14. The idea that psychology is not based on scientific fact or human shortcomings but instead should focus on human experience is the basis for which psychological approach? (A) Cognitive psychology (B) Structuralism (C) Behaviorism (D) Functionalism (E) Humanism

14. (E) Th e basis of humanism is the understanding that individuals have free will and a large capacity for reaching their potential. It is the human experience that we all share that enables individuals to attain such goals. Cognitive psychology is incorrect because it focuses on the process of thinking, perception, and attention to details of language and problem solving. Cognition does not emphasize the human experience. Structuralism focuses on complex mental elements. Behaviorism is based on relationships, stimulus-response, and rewards and punishments. Functionalists examined mental processes, not human experience.

15. Clients who work with their therapists to explore their past to discover the source of their illness would be seeking what type of therapy? (A) Psychoanalytic (B) Humanist (C) Cognitive (D) Eclectic (E) Behavioral

15. (A) Psychoanalysis stresses the importance of the patient and psychologist working together to explore the client's past. Humanism emphasizes one's present and future, not one's past. Cognitive psychology works on changing the client's way of thinking, again not placing much emphasis on the past. Eclectic simply means using several diff erent approaches of psychology. Behavioral psychology tries to identify negative behaviors and eliminate them through such means as systematic desensitization.

16. Psychodynamic psychology focuses mainly on which of the following? (A) Free will and self-actualization (B) Experiments in controlled settings (C) The collective unconscious (D) Thoughts,impulses,anddesiresbeyondtheconsciousbeing (E) Practical introspection

16. (D) Psychodynamic psychology stresses the infl uence of the unconscious. Its fears, impulses, and desires motivate our conscious behavior. Choice (A), free will and selfactualization, refers to humanism. (B) refers to experimental psychology. (C) refers to part of Carl Jung's theory of personality development.

17. A developmental psychologist focuses mainly on: (A) The conscious experiences of an infant (B) The manner in which a child develops the ability to speak, learn, and understand the world around him or her (C) The mental process that helps a young person adapt to his or her environment (D) The identification of one's environment and respond set of the environment (E) Experiments that emphasize actual behavior, rather than controlled settings

17. (B) Developmental psychologists study a person's biological, emotional, cognitive, and social development across the life span. Choice (A) is too vague to be the correct answer. (C) is incorrect because mental process refers to cognitive psychology, not developmental psychology. (D) and (E) are incorrect because they do not answer the question.

18. Phenomenology is best defined as: (A) The study of natural, unanalyzed perception (B) The process of thinking and memory (C) The study of psychological mental health (D) The study of language development (E) The process of consistent patterns and organized sets

18. (A) Choice (A) is the defi nition of phenomenology, the study of natural, unanalyzed perception.

19. The term biological psychology is concerned with: (A) Aggression and sexual behavior (B) Depression and anxiety (C) Genetics and the nervous system (D) Socialanxiety (E) Drug treatment

19. (C) Biological psychologists focus on the ways changes in an organism's physical makeup can aff ect behavior, relating directly to genetics and the nervous system. Choices (A), (B), and (D) are incorrect because biological psychologists do not study the mind or life experiences. Choice (E) may appear to be correct, but the question is asking what the term biological psychology refers to, which is not drug treatment.

20. A case study is: (A) A primary tool for investigation into a client's unconscious through dream analysis and free association (B) A study done over an entire life span of one individual, giving the psychologist detailed information of one's psyche (C) A study that exposes the subject to some event and measures coping skills (D) An independent study used outside the natural environment of the subject (E) A comparative study of various people of different ages at the same time

20. (A) Choice (A) is the defi nition for a case study. Choice (B) defi nes a longitudinal study. Choice (E) defi nes a cross-sectional study. Choices (C) and (D) do not defi ne any type of study

21. Which of the following research methods does not permit researchers to draw conclusions regarding cause-and-effect relationships? (A) Experimental research (B) Surveys (C) Case studies (D) Correlational research (E) Naturalistic observations

21. (D) A correlation expresses a relationship between two variables without ascribing cause. Correlational research employs statistical methods to examine a relationship between two or more variables, but does not permit researchers to draw conclusions. Unlike correlational research, experimental research off ers the opportunity to draw conclusions because of the strict control of variables.

22. A random sample can best be defined as: (A) A sample in which each potential participant has an equal chance of being selected (B) A sample that is carefully chosen so the characteristics of participants correspond to the larger population (C) A selection of cases from a larger population (D) A selection of cases from the control group (E) A sample of a larger population from the experimental group

22. (A) A random sample is defi ned as a sample in which each potential participant has an equal chance of selection. Choice (B) defi nes representative sample. Choice (C) defi nes the term sample, not random sample. Choices (D) and (E) do not accurately defi ne random sample.

23. The Hawthorne effect is best defined as: (A) Expectations by the experimenter that can influence the results of an experiment (B) The change in the results of an experiment when it is "blind" versus "double blind" (C) The idea that people will alter their behavior because of the researchers' attention and not because of actual treatment (D) Specific, test table predictions derived from a theory (E) The idea that subjects in an experiment will lie if the researcher tells them to

23. (C) While researchers were testing the hypothesis that better lighting would boost worker output in an electric plant in the 1920s, they were surprised to see their results showed something else entirely. Productivity increased regardless of lighting merely because of the researcher's attention and not factory conditions. Choice (A) is incorrect because the Hawthorne eff ect focuses on the researcher's attention, not expectations. Choice (B) refers to the researcher's bias and change of behavior, not the subject's.

24. Dr. Bisell conducts an experiment to see whether hunger makes mice run faster through a maze. He randomly assigns 25 mice to a control group or an experimental group. Which cannot be a confounding variable? (A) Where the experiment takes place (B) How hungry the mice were before the experiment (C) How fast the mice are before the race (D) When the experiment takes place (E) The population from which he selected the mice

24. (E) A confounding variable is anything that diff ers between the control group and the experimental group besides the independent variable. How fast and hungry the mice are at the beginning of the experiment are potential confounding variables. When and where the race takes place are also possible confounding variables that can potentially change the fi ndings of this experiment. Th e population from which the mice were selected cannot be a confounding variable. Th is will not diff er for the two groups. All of the mice were chosen from the same larger population. Even if this larger population is fl awed, it is not considered a confounding variable.

25. Marc, a psychology major, collected survey data about the number of hours that college students study for finals and their grades on those finals. His data indicates that students who spend more time studying for finals tend to do better than other students. What can Marc now conclude? (A) Studying improves a student's grade on a final exam. (B) A relationship exists between studying and exam grades. (C) A significant relationship exists between studying and grades. (D) Students who do not study for final exams will not do well on those exams. (E) Students with higher IQs tend to study more than those with lower IQs.

25. (B) Marc has established a relationship. Marc did not conduct an experiment; therefore, he cannot draw any conclusions. Marc has found a correlation between studying and performance on a fi nal exam; whether or not it is signifi cant would require the use of inferential statistics.

26. Jordan runs an experiment testing the effects of sugar consumption on aggression levels in children. He randomly assigns 20 subjects either to a control group given sugar-free candy or to the experimental group that was given the same candy that did contain sugar. He then tests the subjects' response to several different puzzles, each with increasing difficulty. Jordan hypothesizes that sugar levels do play a role in aggression in children. In order to know whether his hypothesis has been supported, Jordan will need to use: (A) Descriptive statistics (B) Means-to-end statistics (C) Experimental research (D) Scatterplots (E) Inferential statistics

26. (E) Jordan would need to use inferential statistics to determine whether the experimental group's aggression levels were signifi cantly diff erent. Jordan could very well use descriptive statistics, but not before he determines whether his hypothesis has been supported and represents the larger population.

27. Which of the following coefficients of a correlation indicate the weakest relationship between two variables? (A) 0.51 (B) −0.28 (C) 0.08 (D) −1.00 (E) 1.00

27. (C) Correlational research allows the researcher to determine whether a relationship exists between two variables. A positive correlation means that high scores on one variable tend to be paired with high scores on the other variable. A number between −1 and +1 expresses the strength of the correlation. A negative correlation means that high scores on one variable tend to be paired with low scores on the other variable. Th e number 0 denotes the weakest possible correlation or no correlation at all.

28. The observation in a classroom that the higher the room temperature, the lower student performance would be an example of: (A) Negative correlation (B) Zero correlation (C) Positive correlation (D) Independent correlation (E) Dependent correlation

28. (A) A negative correlation is expressed as −1. Th is means that as one variable goes up, the other variable will go down. In this case, as the room temperature went up, the student performance went down, indicating a negative correlation.

29. In an experiment, Sydney is going to investigate how alcohol affects aggression. The number of alcoholic drinks the subject has is called: (A) Controlled variable (B) Independent variable (C) Dependent variable (D) Experimental variable (E) Positive variable

29. (B) Th e independent variable in the experiment is the variable that is manipulated to test its eff ects on the other, dependent variables. In this experiment, the manipulation of the number of alcoholic drinks given to the subjects will aff ect their levels of aggression. Th e dependent variable in the experiment is measured to see how it is changed from the manipulation of the independent variable.

30. If a researcher is trying to establish a causal relationship between eating breakfast and work performance, the researcher should use which of the following methods of research? (A) Case study (B) Correlational research (C) Experimental research (D) Survey (E) Statistics

30. (C) With experimental research the strict control of variables off ers the researcher the opportunity to draw conclusions about cause-and-eff ect relationships. In this instance, if the researcher wants to establish a causal relationship between eating breakfast and work performance, experimental research must be used. Correlational research does not allow the researcher to draw conclusions. Surveys simply allow the researcher to gather an immense amount of data in a short period of time.

31. Which part of the brain is responsible for combining sounds into words and arranging words into meaningful sentences? (A) Broca's area (B) Wernicke's area (C) Hypothalamus (D) Hippocampus (E) Medulla

31. (A) Th e Broca's area is located in the left frontal lobe. It is necessary for combining sounds into words and arranging words into meaningful sentences. Wernicke's area plays a role in understanding speech. Th e hypothalamus is part of the limbic system and regulates motivational and emotional behavior. Th e hippocampus is involved in transferring fl eeting memories into permanent storage. Th e medulla is responsible for heart rate and blood pressure.

32. Damage to the cerebellum would most likely result in: (A) Respiratory failure (B) Heart failure (C) Loss of muscular coordination (D) Loss of hearing (E) Loss of memory

32. (C) Th e cerebellum is a region of the hindbrain that is involved in motor control and coordinating movements. Damage to this region would therefore cause loss of muscular coordination.

33. The pons is located between the medulla and other brain areas. It is responsible for which of the following? (A) Motor coordination (B) Seeing and hearing (C) Sleep and arousal (D) Balance (E) Emotional reactions

33. (C) Th e pons is a bridge that connects the spinal cord to the brain. Cells in the pons manufacture chemicals involved in sleep.

34. When humans suffer damage to this part of the brain, there can be a lapse into a permanent state of unconsciousness. (A) Temporal lobe (B) Parietal lobe (C) Frontal lobe (D) Cerebrum (E) Reticular formation

34. (E) Th e reticular formation arouses and alerts the forebrain and prepares it to receive information from all other senses. Damage to this location can cause permanent unconsciousness. Damage to the temporal lobe can cause speech and language issues. Damage to the frontal lobe can cause motivational and emotional issues. Damage to the parietal lobe can cause sensory motor issues.

35. An EEG records: (A) Direct electrical stimulation of the brain (B) The number of neurons in the brain (C) Electrical impulses from the brain (D) Chemicalactivityinspecificareasofthebrain (E) Stimulation of the frontal lobe

35. (C) By measuring electrical impulses, an EEG (electro-encephalogram) can detect epileptic seizures, covert processing, seizure disorders, and sleep disorders.

36. Which part of the brain is affected during a split-brain operation? (A) Cerebellum (B) Corpus callosum (C) Cerebrum (D) Medulla (E) Pons

36. (B) Th e corpus callosum is a wide band of fi bers that connect the left and the right hemispheres of the brain. It has 200 million neural fi bers that allow information to pass back and forth between the hemispheres. It was believed that by cutting the corpus callosum, in what was known as a "split-brain" operation, people suff ering from epilepsy could decrease the number of seizures they had.

37. The limbic system is responsible for (A) The control of hunger, thirst, and sex (B) Breathing regulations (C) Balance and coordination (D) Speech (E) Language

37. (A) Th e limbic system is a group of about half a dozen interconnected structures in the core of the forebrain that are involved in many motivational behaviors, such as eating, drinking, and sexual desire. Breathing regulations are controlled by the medulla. Th e cerebellum controls balance and coordination. Various regions in the left hemisphere of the brain control speech and language.

38. The main job of the thalamus is: (A) Receiving sensory information and relaying it to the appropriate area (B) Processing sensory information about touch, pain, and temperature (C) Regulating motivational and emotional behavior (D) Coordinatingmovementsandtimedmotorresponses (E) Controlling all auditory functions of the brain

38. (A) Th e thalamus is often referred to as the "switchboard" of the brain. All sensory information that enters the brain goes through the thalamus. It is the job of the thalamus to relay the information to the appropriate region of the brain.

39. Bodily sensations such as touch, pressure, and temperature are controlled in which area of the brain? (A) Occipital lobe (B) Temporal lobe (C) Frontal lobe (D) Parietallobe (E) Motor lobe

39. (D) Th e parietal lobe is located directly behind the frontal lobe. Its functions include processing sensory information from the body parts, which includes touching, locating limb positions, and feeling temperature. Th e occipital lobe is responsible for processing visual information. Th e temporal lobe is responsible for processing auditory information. Th e frontal lobe is responsible for interpreting and performing emotional behavior, behaving normally in social situations, and maintaining a healthy personality.

40. As a result of her car accident, Mimi suffered damage to her Broca's area of the brain. What symptoms will she suffer as a result? (A) Inability to see color (B) Inability to speak in fluent sentences (C) Inability to walk (D) Inability to remember short term (E) Inability to remember long term

40. (B) Damage to the Broca's area will result in Broca's aphasia, which means a person cannot speak in fl uent sentences but can understand written and spoken words.

41. If damage occurs to the occipital lobe, an individual could fail to recognize some objects, persons, or color. This damage is called: (A) Visual aphasia (B) Visual agnosia (C) Neglect syndrome (D) Occipital agnosia (E) Temporal aphasia

41. (B) Th e occipital lobe is critical for recognizing objects. Damage to this area results in diffi culties of recognition, a condition called visual agnosia. In visual agnosia the individual fails to recognize some object, person, or color, yet has the ability to see and describe parts of some visual stimuli.

42. A "split-brain" patient is asked to stare at a black dot between the HE and ART as the word HEART is displayed on a screen. When asked what she sees, what will the patient do? (A) The patient will say she sees the word HE. (B) The patient will say she sees the word ART. (C) The patient will point to the word ART. (D) The patient will say the word HEART. (E) The patient will only see a black dot.

42. (B) Th e patient will be able to say she saw the word ART because it was projected to the left hemisphere, which has the ability to control speech. Although the patient's right hemisphere saw the word HE, the right hemisphere turns out to be mute, meaning that it cannot say what it saw. However, the patient can point with her left hand to a photo of HE, indicating the right hemisphere understood the question.

43. Knowing what you are touching or how hot to make the water for your shower involves which of these areas of the brain? (A) Temporal lobe (B) Motor cortex (C) Cerebrum (D) Frontallobe (E) Somatosensory cortex

43. (E) The somatosensory cortex is a narrow strip of the cortex that is located at the front edge of the parietal lobe. It processes sensory information about touch, location of limbs, pain, and temperature.

44. Emma is telling her younger sister stories about her first Christmas in their new home. Which part of the brain is Emma using to recall these memories? (A) Hypothalamus (B) Thalamus (C) Amygdala (D) Hippocampus (E) Medulla

44. (C) The amygdala is involved in forming, recognizing, and remembering emotional experiences, unlike the hippocampus, which is responsible for transferring fleeting memories into permanent storage.

45. An MRI involves: (A) Passing nonharmful radio frequencies through the brain to study brain structure (B) Injecting a slightly radioactive solution into the bloodstream to measure the amount absorbed by the brain (C) Mapping the brain's activity by having the patient complete cognitive tasks (D) Followingbrainimagestogetanexactmeasurementofbrainsize, capacity, and abilities (E) Testing patients' brain damage after severe brain injuries

45. (A) An MRI, or magnetic resonance imaging, involves passing nonharmful radio frequencies through the brain. A PET scan, or positron emission tomography, involves injecting slightly radioactive solutions into the bloodstream.

46. Maddie is walking down a dark alley by herself late at night. She automatically turns her head to the left when she hears a strange noise. What part of the brain is she using? (A) Hindbrain (B) Midbrain (C) Forebrain (D) Somatosensorycortex (E) Motor cortex

46. (B) Th e midbrain is involved in visual and auditory refl exes, such as automatically turning your head toward a noise. Th e hindbrain has three distinct structures: the pons, the medulla, and the cerebellum. Th e forebrain is responsible for a large number of functions, including learning and memory. Th e motor cortex is involved in the initiation of all voluntary movements.

47. Dylan has recovered from extensive injury to his left cerebral hemisphere and has continued his career. His occupation is most likely: (A) Accountant (B) English teacher (C) Journalist (D) Lawyer (E) Graphic artist

47. (E) Choice (E) is the only career that needs some amount of creativity, which is controlled by the right hemisphere. Th e other choices are all careers that need strong language, logical reasoning, and writing skills. Th e left hemisphere controls these skills. Damage to the left hemisphere would make those careers diffi cult.

48. Which of the following is not controlled by the hypothalamus? (A) Sex (B) Eating and drinking (C) Balance and coordination (D) Motivation (E) Emotion

48. (C) Balance and coordination are controlled by the cerebellum. All of the other choices are controlled by the hypothalamus.

49. Which of the following is not part of the limbic system? (A) Hypothalamus (B) Thalamus (C) Cerebellum (D) Amygdala (E) Hippocampus

49. (C) Th e limbic system is a group of structures in the forebrain that are involved in motivational behavior. Th e four structures that make up the limbic system are the hippocampus, hypothalamus, thalamus, and amygdala.

50. Wernicke's area is located on which lobe of the brain? (A) Left temporal lobe (B) Right temporal lobe (C) Left occipital lobe (D) Right occipital lobe (E) Left frontal lobe

50. (A) Th e Wernicke's area is located in the left temporal lobe. Th is area plays a role in understanding speech.

51. Which part of the neuron serves as the protective coating? (A) Axon (B) Dendrite (C) Synapse (D) Myelin sheath (E) Cell body

51. (D) Th e myelin sheath is composed of fatty material that wraps around and insulates an axon. Th e axon is a single threadlike structure that carries signals away from the cell body. Th e dendrites are branchlike extensions that arise from the cell body. Th e synapse is a small space that exists between an end bulb and adjacent cell body. Th e cell body provides fuel and maintains the neuron.

52. Another name for the cell body of the neuron is: (A) Dendrite (B) Myelin (C) Soma (D) Axon (E) Synaptic vesicle

52. (C) Another name for the cell body is the soma, a relatively large structure that maintains the entire neuron

53. The process by which a tiny electrical current is generated when the positive sodium ions rush inside the axon, causing the inside of the axon to reverse its charge, is called: (A) Action potential (B) Ion potential (C) Resting state (D) Synapticstate (E) Negative potential

53. (A) If a stimulus is large enough to excite a neuron, two things will happen to the axon. First the stimulus will eventually open the axon's chemical gates by stopping the sodium pump. Second, when the stoppage of the sodium pump causes the gate to open, thousands of positive ions will rush in. Th e action potential is a tiny electrical current that is generated when positive sodium ions rush into the axon. A resting state is when the axon has a charge, like a battery, with positive ions on the outside and negative ions on the inside.

54. If Mia stepped on a nail, which of the following would be the correct order of communication for her to feel the pain? (A) Stimulus-electrical impulse-neurotransmitter-receptor site (B) Electrical impulse-stimulus-receptor site-neurotransmitter (C) Receptor site-neurotransmitter-electrical impulse-stimulus (D) Electricalimpulse-receptorsite-stimulus-neurotransmitter (E) Stimulus-electrical impulse-receptor site-neurotransmitter

54. (A) When you step on a sharp object, you seem to feel the pain almost immediately. Neurons send signals at speeds as high as 200 miles per hour. To feel the pain involves several events happening in this order: Th e stimulus—in this example, stepping on a nail— begins the reaction. Sensors in your skin then pick up the mechanical pressure and transforms it into an electrical impulse. When the impulse reaches the end bulb it releases the neurotransmitter, which is the chemical messenger that transmits information between nerves and body organs. Since the stimulus must come fi rst, choices (B), (C), and (D) can be eliminated. Choice (E) is incorrect because the neurotransmitter has to be released before anything can reach the receptor site.

55. What is the job of the sodium pump? (A) It separates positive ions and places them all inside the axon. (B) It is responsible for keeping the axon charged by returning and keeping sodium ions outside the axon membrane. (C) It generates an electrical current when the positive ions rush into the axon. (D) It generates an electrical current when the negative ions rush in to the axon. (E) It is a neural impulse that transfers negative ions into the neuron.

55. (B) Th e sodium pump is a transport process that picks up any sodium ions that enter the axon's chemical gates and returns them back outside. Choice (A) is incorrect because when the axon is charged, positive ions are on the outside while negative ions are on the inside. Choices (C) and (D) do not correctly defi ne a sodium pump. Choice (E) is incorrect because the sodium pump is not a neural impulse

56. If an action potential starts at the beginning of an axon, the action potential will continue at the same speed to the very end of the axon. This concept is known as: (A) Nerve impulse (B) Synapse (C) Resting state (D) All-or-nonelaw (E) Sodium pump

56. (D) Th e all-or-none law is the principle that the action potential in a neuron does not vary in strength; the neuron either fi res at full strength or it does not fi re at all. Choice (B) is incorrect because the synapse is the area composed of the axon terminal of one neuron and the dendrite of the next neuron. Choice (C) is incorrect because the resting state is when a neuron is positively charged outside and negatively charged on the inside. Choice (E) is incorrect because the sodium pump is a transport process that picks up sodium ions.

57. Which of the following functions best explains the role of the sympathetic nervous system? (A) Preparing the body for a traumatic event (B) Returning the body to equilibrium (C) Preparing the body for "fight or flight" (D) Maintainingthebody'svitalfunctions (E) Maintaining homeostasis

57. (C) Th reatening or challenging physical or psychological stimuli triggers the sympathetic nervous system. Th is increases physiological arousal and prepares the body for action. Th e sympathetic nervous system prepares the body for "fi ght or fl ight." Th e parasympathetic nervous system helps return the body to equilibrium, also called homeostasis.

58. Which of the following neurotransmitters most closely resembles the affects alcohol has on the nervous system? (A) Anandamide (B) GABA (C) Dopamine (D) Acetylcholine (E) Serotonin

58. (B) Alcohol aff ects the nervous system in a number of ways, blocking neural receptors and stimulating others. Some neurons are excited by the neurotransmitter GABA, which the brain normally manufactures. Alcohol molecules so closely resemble those of GABA neurotransmitters that alcohol can function like GABA and open GABA receptors. Anandamide is involved in memory, motor coordination, and emotions. Dopamine is critical to the way the brain controls movement; there is a direct link to dopamine levels in the body and Parkinson's disease and schizophrenia. Acetylcholine is a major excitatory neurotransmitter. Serotonin infl uences mood levels in the body.

59. What is one major difference between the sympathetic and parasympathetic nervous systems? (A) The sympathetic nervous system increases physiological arousal, while the parasympathetic nervous system returns the body to a calmer and relaxed state. (B) The sympathetic nervous system is a subdivision of the somatic nervous system, while the parasympathetic nervous system is a subdivision of the autonomic nervous system. (C) The sympathetic nervous system plays a role in traumatic events, while the parasympathetic nervous system only plays a role in digestion. (D) The parasympathetic nervous system is used more often than the sympathetic nervous system. (E) The sympathetic nervous system plays a role in sexual behavior, while the parasympathetic nervous system does not.

59. (A) Th e sympathetic nervous system and parasympathetic nervous system are both subdivisions of the autonomic nervous system. Th e sympathetic nervous system prepares the body for threatening or challenging situations, which means increased blood pressure and increased heart rate. Th e parasympathetic nervous system returns the body to a relaxed state, for example, decreased heart rate

60. Neurons that carry information away from the spinal cord to produce responses in various muscles or organs throughout the body are called: (A) Afferent neurons (B) Interneurons (C) Neuro transmitters (D) Sensorneurons (E) Efferent neurons

60. (E) Efferent neurons carry information away from the spinal cord to produce responses in various muscles. Afferent neurons carry information from the senses to the spinal cord. Interneurons carry information within the central nervous system.

61. The basic experience of the stimulation of the body's senses is called: (A) Sensation (B) Perception (C) Adaptation (D) Cognition (E) Conduction

61. (A) Sensation is the experience of sensory stimulation. Perception is the process of creating meaningful patterns from the sensory information. Adaptation is the decreasing response of the sense organs upon exposure to a continual stimulation.

62. Taste: 1 gram of table salt in 500 liters of water, smell: 1 drop of perfume diffused throughout a three-room apartment, touch: the wing of a bee falling on your cheek from a height of 1 centimeter away. These are all examples of: (A) The just-noticeable difference of our senses (B) The difference threshold for our senses (C) The absolute threshold of our senses (D) The adaptation of our senses (E) The perception of our senses

62. (C) Th e minimum intensity of physical energy required to produce any sensation at all in a person is called absolute threshold. Th e diff erence threshold, also known as the justnoticeable diff erence, is the smallest change in stimulation that can be detected 50 percent of the time

63. Weber's law can best be defined as: (A) The smallest change in stimulation that can be detected 50 percent of the time (B) The principle that the just-noticeable difference for any given sense is a constant proportion of the stimulation being judged (C) The principle that there is an adjustment of sensation levels depending on the stimulation received (D) The idea that the least amount of energy detected in a stimulation only occurs 50 percent of the time (E) The theory that all stimuli respond to the same sensations through the process of creating meaningful patterns

63. (B) Weber's law states that the JND (just-noticeable diff erence) for any given sense is a proportion of the stimulation being judged. Hearing, for example, is very sensitive: we can detect a 0.3 percent change in sound. By contrast, producing a JND in taste requires a 20 percent change.

64. The name of the transparent protective coating over the front part of the eye is: (A) Lens (B) Iris (C) Pupil (D) Fovea (E) Cornea

64. (E) Th e transparent protective coating over the front part of the eye is the cornea. Th e lens focuses the light onto the retina. Th e iris is the colored part of the eye. Th e pupil is the small opening in the iris where light enters. Th e fovea is the area of the retina that is the center of the visual fi eld.

65. The function of the lens is to: (A) Project an image onto the cornea (B) Focus an image on the retina (C) Locate an image (D) Contain receptor cells that are sensitive to light (E) Locate the blind spot

65. (B) Th e lens is the transparent part of the eye inside the pupil that focuses light onto the retina

66. The greatest density of cones exists in which part of the eye? (A) Cornea (B) Lens (C) Pupil (D) Fovea (E) Retina

66. (D) Th e photoreceptors with a conelike shape are called cones. Th ey are primarily located in the center of the retina, called the fovea. Th e fovea is the correct answer, and not the retina, because the question was looking for the location of the greatest density of cones.

67. An afterimage can best be defined as: (A) Sense experience that occurs after a visual stimulus has been removed (B) Decreased sensitivity of rods and cones in bright light (C) Increased sensitivity of rods and cones in darkness (D) Distinguish able fine details of a stimulation (E) Non-distinguishable details of a stimulation

67. (A) An afterimage is a visual sensation that continues after the original stimulus is removed. For example, if you stare at a blue square, you will see a yellow afterimage.

68. The theory of color that best explains color afterimage is: (A) The volley theory (B) The trichromatic theory (C) The opponent-process theory (D) The subtractive color theory (E) The monochromatic theory

68. (C) On the basis of his work with afterimages, physiologist Ewald Hering suggested that the visual system codes colors by using two complementary pairs: red/green and blue/ yellow. Hering's idea became known as the opponent-process theory. Th e trichromatic theory says there are three diff erent kinds of cones in the retina, not related to an afterimage. Th e volley principle has to do with receptors in the ear and has no relation to an afterimage.

69. Trichromats can mix which three colors to perceive virtually any hue? (A) Red, blue, green (B) Red, blue, yellow (C) Blue, yellow, green (D) Red,green,yellow (E) Yellow, orange, green

69. (B) Trichromats are people who have normal color vision. Trichromats perceive all hues by combining the colors red, blue, and green.

70. The three small bones of the inner ear are called what? (A) Cochlear bones (B) Tympanic bones (C) Basilar (D) Ossicles (E) Auditory canals

70. (D) Th e three small bones are called the hammer, anvil, and stirrup, also known as the ossicles.

71. When the molecules of a skunk's spray enter your nose, the molecules are transformed into electrical signals, or impulses, that are interpreted by the brain as an unpleasant odor. This is an example of: (A) Adaptation (B) Transduction (C) Sensation (D) Perception (E) Stimulation

71. (B) Transduction refers to the process in which a sense organ, in this case the nose, changes or transforms physical energy into electrical signals that become neural impulses, which may be sent to the brain for processing. Choice (A) is incorrect because adaptation refers to a decreased response to a stimulation. Choice (C) is incorrect because sensation is a meaningless bit of information. Choice (D) is incorrect because perception is meaningful sensory experiences.

72. Which of the following occupations relies heavily on kinesthetic and vestibular senses? (A) Doctor (B) Pilot (C) Gymnast (D) Artist (E) Engineer

72. (C) A gymnast relies on both her kinesthetic and her vestibular senses. Her kinesthetic senses are relaying messages pertaining to muscle strain and movements; her vestibular senses are supplying feedback about her body position. Kinesthetic senses are senses of muscle movement, posture, and strain on muscles and joints. Vestibular senses are the senses of equilibrium and body position.

73. Frequency is to as amplitude is to . (A) sensation; perception (B) loudness; pitch (C) pitch; loudness (D) perception; sensation (E) warmth; cold

73. (B) Loudness is our subjective experience of a sound's intensity. Th e brain calculates loudness from specifi c physical energy, in this case the amplitude of sound waves. Pitch is our subjective experience of a sound being high or low. Th e frequency of the sound wave is measured in cycles.

74. Olfactory cells are the receptors for what sense? (A) Taste (B) Hearing (C) Vision (D) Smell (E) Touch

74. (D) Th e olfactory cells are located in two one-inch-square patches of tissue in the uppermost part of the nasal passages.

75. The binocular cue for depth perception based on signals from muscles that turn the eyes to focus on near or approaching objects is called: (A) Convergence (B) Retinal disparity (C) Shape constancy (D) Interposition (E) Perceptual vision

75. (A) Convergence is a binocular cue for depth perception based on signals sent from muscles that turn the eye. To focus on near or approaching objects, these muscles turn the eyes inward, toward the nose. Retinal disparity refers to the diff erent position of the eyes receiving slightly diff erent images. Shape constancy refers to the tendency to perceive an object as retaining the same shape even when you view it from diff erent angles. Interposition comes into play when objects overlap

76. As a car drives away, it projects a smaller and smaller image on your retina. Although the retinal image grows smaller, you do not perceive the car as shrinking because of: (A) Shape constancy (B) Size continuity (C) Size constancy (D) Shapecontinuity (E) Size perception

76. (C) Size constancy refers to our tendency to perceive objects as remaining the same size even when their images on the retina are continually growing or shrinking. Choice (A), shape constancy, refers to changing shapes, not necessarily size.

77. Which of the following is not a monocular depth cue? (A) Linear perspective (B) Interposition (C) Relative size (D) Texturegradient (E) Convergence

77. (E) Convergence is a binocular cue, meaning the cue depends on the movement of both eyes. Choices (A), (B), (C), and (D) are monocular cues, that is, cues that are produced from a single eye

78. The final step required to convert vibrations into sound sensations takes place in which part of the ear? (A) Ossicles (B) Outer ear (C) Cochlea (D) Middleear (E) Auditory receptors

78. (C) Th e cochlea is located in the inner ear. Th e cochlea contains the receptors for hearing, and its function is transduction, transforming vibrations into nerve impulses that are sent to the brain for processing into auditory information.

79. Which of the following statements best defines the gate control theory of pain? (A) Pain impulses are sent to receptor sites in vital organs. (B) Nonpainful nerve impulses compete with pain impulses to reach the brain, creating a neural blockage. (C) Stimuli of various kinds activate free nerve endings. (D) Pain is simply a psychological state, not a physiological one. (E) Perception of pain depends on one's physical makeup.

79. (B) Th e gate control theory explains that you may not notice pain from a headache or injury while thoroughly involved in some other activity, because impulses from that activity close the neural gate and block the passage of painful impulses

80. Black-and-white vision with greatest sensitivity under low levels of illumination describes the role of: (A) The cones (B) The cornea (C) The fovea (D) Therods (E) The pupil

80. (D) Rods are photoreceptors that contain a single chemical, called rhodopsin, which is activated by small amounts of light. Because rods are extremely light sensitive, they allow us to see in dim light, but to see only black, white, and shades of gray. Cones are photoreceptors that contain three chemicals called opsins, which are activated in bright light and allow us to see color.

81. Which of the following is not considered to be an altered state of consciousness? (A) Sleep (B) Hypnosis (C) Psychoactive drugs (D) Exercise (E) Meditation

81. (D) Altered states of consciousness result from using any number of procedures, such as meditation, psychoactive drugs, hypnosis, or sleep deprivation. Choices (A), (B), (C), and (E) all diff er from normal consciousness. Th e chief characteristic of these altered states, unlike exercise, is that we perceive our internal and external environments in ways diff erent from normal perception.

82. Driving a car along a familiar route while listening to the radio or thinking of something else is an example of: (A) Automatic process (B) Controlled process (C) Somatic process (D) Sympatheticprocess (E) Parasympathetic process

82. (A) Th e automatic process is any activity that requires little awareness, takes minimal attention, and does not interfere with ongoing activities. All of these characteristics describe what sometimes happens while people are driving a familiar route. Choice (B) requires full awareness. Choices (C), (D), and (E) do not pertain to this question.

83. When researchers removed all time cues, such as light, clock, radio, and television, from subjects' environment, the length of the day expanded from 24 to about 25 hours. This phenomenon is known as: (A) The interval timing clock (B) The circadian rhythm (C) The biological clock (D) The internal rhythm (E) The external clock

83. (B) Th e circadian rhythm refers to a biological clock that is genetically programmed to regulate physiological responses within a time period of 24-25 hours (one day). Most of us operate on a 24-hour day and thus set back our sleep-wake circadian clock about one hour each day. Choice (A), interval timing clock, works more like a stopwatch, which helps a person to time his or her movements, such as knowing when to start or stop an activity. Choice (C), biological clock, is an internal timing device used to regulate various physiological responses, but it is not genetically programmed.

84. The hormone most closely related to one's sleep patterns is: (A) Serotonin (B) Norepinephrine (C) Epinephrine (D) Melatonin (E) Dopamine

84. (D) Melatonin is a hormone that is secreted by the pineal gland. Melatonin secretion increases with darkness and decreases with light, playing a role in the regulation of circadian rhythms and in promoting sleep. Serotonin is related to mood levels and mood control. Norepinephrine works as a stress hormone and is directly related to "fi ght or fl ight." Epinephrine, when produced by the body, increases heart rate and blood pressure. Dopamine also relates to the sympathetic nervous system, increasing heart rate and blood pressure.

85. The sleep stage that is a transition from wakefulness to sleep and lasting 1-7 minutes is: (A) REM sleep (B) Stage 1 sleep (C) Stage 2 sleep (D) Stage3sleep (E) Stage 4 sleep

85. (B) Stage 1 sleep is a transition stage from wakefulness to sleep. In this stage a person gradually loses responsiveness to stimuli and experiences drifting thoughts and images. REM sleep, or paradoxical sleep, is marked by physiological arousal and voluntary muscle paralysis. Stage 2 sleep marks the beginning of a deeper sleep. Stages 3 and 4 are characterized by low-frequency waves; stage 4 specifi cally is considered to be the deepest sleep stage.

86. Which stage of sleep is characterized by delta waves (very high amplitude and very low frequency)? (A) Stage 4 sleep (B) Stage 3 sleep (C) Stage 2 sleep (D) Stage1sleep (E) REM sleep

86. (A) Stage 4 sleep is also called slow wave, or delta, sleep. It is characterized by waves of very high amplitude and low frequency, called delta waves.

87. When in this stage of sleep, brain waves have a fast frequency and low amplitude and look very similar to beta waves, which occur when you are wide-awake and alert. Which state of sleep is this? (A) Stage 1 sleep (B) Stage 2 sleep (C) Stage 3 sleep (D) REMsleep (E) Stage 4 sleep

87. (D) REM sleep is also known as paradoxical sleep. REM brain waves have fast frequency and low amplitude and look very similar to beta waves, which occur when you are wide-awake. During this stage your body is physiologically aroused, but your voluntary muscles are paralyzed. REM sleep stage is highly associated with dreaming.

88. Sleepwalking and sleep talking are characteristics of which stage of sleep? (A) Stage 1 sleep (B) Stage 2 sleep (C) Stage 3 sleep (D) Stage4sleep (E) REM sleep

88. (D) Sleepwalking and sleep talking do occur during stage 4 sleep. Many people confuse this answer with REM stage, because of the belief that sleepwalkers and sleep talkers are acting out their dreams that occur in REM. But voluntary muscles are paralyzed during REM; therefore, people cannot physically act out their dreams. Because stage 4 is the deepest stage of sleep, very often people do not remember sleepwalking or sleep talking.

89. An infant sleeps approximately 17 hours a day. Of those hours, how many are spent in REM? (A) 20 percent (B) 30 percent (C) 50 percent (D) 70percent (E) 80 percent

89. (C) From infancy to adolescence, the total amount of time spent in sleep and the percentage spent in REM gradually decline. Newborns sleep about 17 hours a day, and 50 percent of that time is spent in REM. A four-year-old sleeps about 10 hours, and 25 percent of that time is spent in REM. From adolescence to old age, we maintain the same amount of sleep time, approximately 7.5 hours of sleep, and the same percentage of REM sleep, about 20 percent or less.

90. The adaptive sleep theory suggests: (A) Daily activities deplete key factors in our brain and body that are replenished by sleep. (B) Sleep evolved because it prevented early humans and animals from wasting energy and exposing themselves to dangers of nocturnal predators. (C) For our internal clocks to have synchrony with the external world, thereby decreasing fatigue, disorientation, and lack of concentration, sleep is necessary. (D) Sleep is necessary to combat insomnia and drowsiness. (E) External environments are constantly competing with individual sleep rhythms. Sleep is necessary to compete with the external clock.

90. (B) Choice (B) defi nes the term adaptive sleep theory. Support for the adaptive theory comes from observations that large predatory animals sleep more and wherever they want, while smaller prey sleep less and in more protected areas. Choice (A) defi nes the term repair theory

91. The center of the activation-synthesis hypothesis of dreaming is based on the belief that: (A) The conscious needs to express unfulfilled wishes. (B) Dreams provide an outlet for repressed thoughts. (C) Dreams provide explanations for physiological activity. (D) The unconscious needs to exhibit socially unacceptable behavior (E) Dreams allow the individual to work out daily hassles.

91. (C) Th e activation-synthesis theory of dreams says that dreaming represents the random and meaningless activity of nerve cells in the brain. Choices (A) and (B) represent the Freudian view of dreaming. Choice (E) represents the extension of waking life theory

92. The majority of our dreams occur in which stage of sleep? (A) REM sleep (B) Stage 1 sleep (C) Stage 2 sleep (D) Stage3sleep (E) Stage 4 sleep

92. (A) REM sleep, which stands for "rapid eye movement," is associated with dreaming. Dream research suggests that about 80-90 percent of the times when subjects are awakened from REM sleep, they report having had a vivid and long dream. Only about 5-10 percent of our dreams occur in stage 4 sleep and are less likely to be remembered.

93. The idea that dreams represent wish fulfillment comes from which theory of dream interpretation? (A) Extension of waking life (B) Activation synthesis (C) Spiritual world (D) Transformation dream analysis (E) Freud's theory of dream interpretation

93. (E) Freud's view on dreaming was the belief that dreams protect the conscious from the realization of our unconscious desires and wishes, especially sexual or aggressive wishes. Our dreams transform these desires into harmless symbols and do not disturb our sleep. Extension of waking life is based on the belief that our dreams refl ect the same thoughts and concerns we have when we are awake. Th e activation-synthesis theory suggests that dreams are a product of neural fi rings in our brain. Th e spiritual world theory states that dreams represent the time when one enters the spiritual world, which helps a person to refl ect on the past, present, or future, through communication with the souls of people who are no longer with us.

94. Repeated periods during sleep when a person stops breathing for 10 seconds or longer is known as: (A) Narcolepsy (B) Sleep apnea (C) Sleep agnosia (D) Insomnia (E) Night terrors

94. (B) A person with sleep apnea may repeatedly stop breathing, momentarily wake up, resume breathing, and return to sleep. Narcolepsy is marked by excessive sleepiness usually in the form of sleep attacks. Insomnia refers to diffi culties in either going to sleep or staying asleep through the night.

95. A person experiences blind panic, screaming, and thrashing around while sleeping. This episode is called: (A) A night terror (B) A nightmare (C) A sleep terror (D) Dreaming (E) A REM rebound episode

95. (A) Night terrors are frightening experiences that often start with screaming, followed by sudden waking in a fearful state with rapid breathing. Th ey usually occur in stage 4 sleep. Night terrors are often confused with nightmares, which usually occur during REM sleep. Th ey are also frightening, but usually produce clear anxiety-producing images.

96. A relatively rare condition that involves irresistible attacks of sleepiness, brief periods of REM, and often muscle paralysis is called: (A) Sleep apnea (B) Sleep terror (C) Narcolepsy (D) Benzodiazepines (E) Night terror

96. (C) Narcolepsy is a chronic disorder. It is characterized by sleep attacks or short lapses of sleep throughout the day. Th ese attacks are accompanied by REM sleep and muscle paralysis.

97. REM sleep is also known as paradoxical sleep because: (A) Measures of the brain activity closely resemble waking consciousness, but the person is in the deepest stage of sleep. (B) Measures of the brain activity closely resemble waking consciousness, but the person is incapable of moving. (C) The person's heart rate is slower than when awake, but the person can sleepwalk or sleep talk. (D) The person can have night terrors during this stage but will not remember them in the morning. (E) The person's vital signs are very slow, but the person can get up and walk around.

97. (B) REM sleep looks very similar to beta waves. Physiologically a person is aroused during this stage and muscles are paralyzed, which is why this stage is known as "paradoxical sleep." Choice (A) is incorrect because REM sleep is not the deepest stage of sleep; stage 4 is. Choice (C) is incorrect because body paralysis occurs during REM; therefore, a person cannot sleepwalk. Choice (D) is incorrect because night terrors occur in stage 4 sleep, not REM. Choice (E) is incorrect because a person's vital signs are actually very aroused in REM

98.The mental state that encompasses the thoughts, feelings, and perceptions. that occur when we are reasonably alert is called: (A) Altered state of consciousness (B) Subconscious (C) Preconscious (D) Alert consciousness (E) Waking consciousness

98. (E) Waking consciousness is a mental state that encompasses all thoughts and perceptions that occur when we are awake. Th e altered state of consciousness awareness is diff erent from the consciously awake person. Choices (B) and (C) represent states of mind diff erent from the consciously awake person as well

99.Alteration in consciousness that occurs seemingly without effort, typically when we want to momentarily escape reality, is called: (A) Daydreaming (B) Dreaming (C) Meditation (D) Hypnosis (E) Anesthesia

99. (A) One of the main reasons people daydream is to escape reality. It is usually done without eff ort or recognition. In choices (C), (D), and (E), a person does recognize he or she is doing something to escape, usually with more eff ort. Dreaming, on the other hand, occurs without any recognition.

100.A sleep disorder characterized by difficulty in falling asleep or remaining asleep is called: (A) Narcoplepsy (B) Sleep apnea (C) Insomnia (D) Sleepterror (E) Nightmares

100. (C) Insomnia is diffi culty with either falling asleep or staying asleep. Narcolepsy is a disorder characterized by sleep attacks. Sleep apnea is marked by periods of sleep when a person stops breathing.

101. Which of the following is not a characteristic of REM sleep? (A) Rapid eye movement (B) Vivid dreams (C) Increased heart rate (D) Paralysis (E) Delta waves

101. (E) Delta waves are slow waves with a very high amplitude and very low frequency. Delta waves are part of stage 4 sleep, not REM. All of the other choices are defi nite characteristics of REM sleep

102. Approximately how many cycles of sleep does an adult enter during a full night's sleep? (A) One to two (B) Th ree to four (C) Four to fi ve (D) Six to seven (E) Seven to eight

102. (C) An adult getting approximately seven to eight hours of sleep will go through four to fi ve cycles of sleep. A full cycle begins with stage 1 sleep and ends with REM. Th e next cycle starts at stage 2 and goes up to stage 3 and 4 and back to REM again. Individuals do not return to stage 1 until around the time they are going to wake up

103. Approximately how long is each cycle of sleep during a full night's sleep? (A) 80 minutes (B) 90 minutes (C) 60 minutes (D) 70 minutes (E) 50 minutes

103. (B) Each stage is 90 minutes. Th e fi rst cycle includes stages 1, 2, 3, 4, and REM. Th e next cycle begins with stage 2.

104. Experimenters have shown that a person deprived of the stage of sleep will become anxious, testy, and hungry and have difficulty concentrating. (A) REM (B) Stage 1 (C) Stage 2 (D) Stage 3 (E) Stage 4

104. (A) Many psychologists in the 1950s believed that if people were denied REM sleep and therefore could not dream, they would suff er mentally and emotionally. Studies today continue to show long-term detrimental behavioral problems when people do not get enough REM sleep. Th is is not the case with the other stages of sleep

105. Before entering sleep, you briefl y pass through a relaxed and drowsy state. Th is is marked by which characteristic? (A) Beta waves (B) Delta waves (C) Alpha waves (D) Th eta waves (E) Zeta waves

105. (C) Alpha waves are characteristic of this period before entering sleep. Delta waves are characteristic of stage 4 sleep. Beta waves are characteristics of REM sleep. Th eta waves are characteristic of stage 1 sleep.

106. Which part of the brain is important in keeping the forebrain alert and producing a state of wakefulness? (A) Hippocampus (B) Limbic system (C) Hindbrain (D) Reticular formation (E) Medulla

106. (D) Th e reticular formation arouses and alerts the forebrain. It is stimulated in sleeping animals. Choices (A) and (B) have to do with memory and emotion. Choice (E) has to do with breathing and heart rate.

107. The dream theory that suggests our dreams reflect the same thoughts, fears, and concerns present when we are awake is called: (A) Freud's theory of dreams (B) Extension of waking life (C) Activation-synthesis (D) External world (E) Spiritual world

107. (B) Extension of waking life theory suggests that dreams refl ect our thoughts and concerns from our waking lives, or issues we have on our minds when we are awake. Freud's theory suggests our dreams represent our repressed desires and fantasies. Activation synthesis suggests dreams are a product of our neural fi rings in the brain. Spiritual world theory suggests when we dream we are in touch with those who have passed on.

108. Eighty percent of our sleep takes place in which cycle of sleep? (A) Stage 1 (B) Stage 2 (C) Stage 3 (D) Stage 4 (E) All of the above

108. (E) As an adult, 80 percent of our sleep is in NREM. In other words, adults spend 20 percent of their sleep in REM sleep

109. Beta waves are characteristic of a person who is: (A) Dreaming (B) In a coma (C) Asleep but not dreaming (D) Awake and alert (E) In stage 1 sleep

109. (A) Beta waves are characteristic of REM sleep. REM sleep is where 90 percent of our dreaming occurs; therefore, beta waves are characteristic of a person who is dreaming.

110. refers to an increased percentage of time spent in REM sleep when we are deprived of REM sleep on the previous night. (A) REM rebound (B) REM deprivation (C) REM sleep (D) REM makeup (E) REM extension

110. (A) REM rebound is the idea that we go straight to REM sleep when we are sleep deprived

111. According to Ernest Hilgard's hidden observer theory, people who are hypnotized and told to plunge one hand into a glass of painfully cold ice water with the suggestion they will not feel pain, will respond to the question "Do you feel pain?" by: (A) Saying they do not feel pain (B) Waking up from the hypnotic trance (C) Screaming and removing their hand from the water (D) Screaming but leaving their hand in the water (E) Saying they do feel pain

111. (A) Ernest Hilgard developed the hidden observer concept. Th e idea was that under a hypnotic trance a person's conscious is actually divided into two parts. Th e hypnotized part will feel little or no pain and will respond that way orally. Th e unhypnotized part will feel normal pain sensations but will not answer the question orally. Th is part can respond to the question by tapping one's fi ngers.

112. Which of the following drugs are physically addictive? (A) Morphine (B) Cocaine (C) Heroin (D) All of these (E) None of these

112. (D) Morphine, cocaine, and heroin are all highly physically addictive drugs, causing a person abusing these drugs to feel an overwhelming and compulsive desire to obtain and abuse the drug. Even after stopping, the person has a great tendency to relapse and begin using the drug again.

113. Which statement best defines dependency? (A) The original dosage of the drug no longer produces desired effects. (B) Behavioral patterns are marked by overwhelming desire to obtain and use the drug. (C) A change in the nervous system occurs so that a person now needs to take the drug to prevent withdrawal symptoms. (D) Painful physical and psychological symptoms occur after the drug is no longer in the system. (E) Decompression from the peripheral nervous system begins after the drug enters the body

113. (C) Th e defi nition for dependency is a change in the nervous system that results in a person's needing the drug to prevent painful withdrawal symptoms. Choice (A) defi nes tolerance. Choice (B) defi nes addiction. Choice (D) defi nes withdrawal symptoms.

114. Which of the following drugs block reuptake, leading to increased neural stimulation? (A) Heroin (B) Cocaine (C) Morphine (D) Amphetamines (E) Methamphetamines

114. (B) When excited, neurons secrete neurotransmitters. After a brief period of time, the neurotransmitters are reabsorbed back into the neuron. Th is process is called reuptake. If reuptake did not occur, the neurotransmitter would remain in the synapse and neurons would be continually stimulated. Cocaine blocks reuptake, which leads to increased neural stimulation, causing increased physical and psychological arousal.

115. Which of the following drugs does not fall under the category of a stimulant? (A) Cocaine (B) Caff eine (C) Nicotine (D) Amphetamines (E) Heroin

115. (E) Stimulants, by defi nition, increase activities of the central nervous system. Th is results in heightened alertness, arousal, and euphoria. Cocaine, caff eine, nicotine, and amphetamines are all stimulants. Heroin is an opiate, which is highly addictive and used for pain reduction.

116. The reduction in the body's response to a drug, which may accompany continual drug use, is called: (A) Withdrawal (B) Addiction (C) Dependency (D) Tolerance (E) Hallucinations

116. (D) Tolerance occurs after a person uses a drug repeatedly over a period of time. Th e drug no longer produces the desired eff ects. Withdrawal is the painful symptoms that occur when a person is no longer taking an addictive drug. Addiction is the behavioral pattern marked by a compulsive desire for the drug.

117. A teenage boy once described using this drug as "life without anxiety, . . .it makes you feel good." However, this boy eventually discovered the dark side of the drug. With constant use, dosages became larger and larger. Eventually getting high was almost impossible and normal functioning was out of the question. Which drug was he referring to? (A) Cocaine (B) Nicotine (C) Heroin (D) LSD (E) Psilocybin

117. (C) Cravings for heroin, unlike other drugs, become very intense very quickly. During detoxifi cation, a person can suff er from vomiting, nausea, diarrhea, and chills. Th is is part of the reason why heroin is such a powerful drug. Although the other choices are also highly addictive drugs, heroin has such severe withdrawal symptoms that it is much harder to stop.

118. Hallucinogens are best defi ned as: (A) Psychoactive drugs that produce strange and unusual perceptual, sensory, and cognitive experiences (B) Stimulants that produce arousals both physically and psychologically (C) Designer drugs that cause three primary eff ects, pain reduction, euphoria, and tolerance (D) Mild depressants that decrease heart rate and blood pressure (E) Drugs that stimulate the central nervous system

118. (A) Hallucinogens are a separate category from stimulants, eliminating choices (B) and (E). Choice (C) describes some characteristics of opiates. Hallucinogens are not depres- sants, thereby eliminating choice (D).

119. In order for a person to be hypnotized, the hypnotist must do which of the following? (A) Suggest what the subject will experience during hypnosis (B) Tell the subject what he or she will be doing while under hypnosis (C) Tell the subject to count from ten to one (D) Suggest that the subject enter a trance (E) Tell the subject to relax and feel no stress

D

120. Which age group of people is most susceptible to hypnosis? (A) 20-24 (B) 17-20 (C) 15-19 (D) 8-12 (E) 45-49

D

121. Cold sweats, vomiting, convulsions, and hallucinations are all symptoms of what drug? (A) LSD (B) Cocaine (C) Methamphetamines (D) Barbiturates (E) Heroin

A

122. are psychoactive drugs that depress the central nervous system, while stimulate the central nervous system. (A) Opiates, barbiturates (B) Opiates, amphetamines (C) Barbiturates, amphetamines (D) Amphetamines, barbiturates (E) Amphetamines, opiates

C

123. What are the four major areas of impact of psychoactive drugs? (A) Appetite, behavior, sex drive, and perception (B) Perception, behavior, moods, mental processes (C) Perception, mental processes, appetite, digestion (D) Appetite, perception, moods, mental processes (E) Mental processes, moods, digestion, perception

B

124. Which of the following psychoactive drugs is not a depressant? (A) Alcohol (B) Barbiturates (C) Benzodiazepines (D) Heroin (E) Nembutal

D

125. Th is drug induces a number of physiological and psychological eff ects, some of which include dilated blood vessels in the eye, dry mouth, time distortion, euphoric feelings, sense of relaxation, and mild muscular weakness. (A) Alcohol (B) Marijuana (C) LSD (D) Tranquilizers (E) Cocaine

B

126. Which of the following is not a practical application of hypnosis? (A) Ease pain (B) Stop smoking (C) Remember a painful event (D) Stop overeating (E) Marriage counseling

E

127. In the 1700s a force called "animal magnetism," later known as hypnosis, was introduced by: (A) Sigmund Freud (B) Ernest Hilgard (C) Wilhelm Wundt (D) William James (E) Anton Mesmer

E

128. In using hypnosis for pain reduction, patients highly susceptible to hypnosis were: (A) More likely to experience posthypnotic amnesia (B) Less likely to participate in future studies (C) More likely to report significantly lower pain levels (D) Less likely to report lower pain levels (E) Likely to respond more slowly to the induction method

c

129. All of the following are terms related to hypnosis except: (A) Posthypnotic amnesia (B) Hidden observer (C) Suggestibility (D) Hypnotic analgesia (E) Posthypnotic exhortation

e

130. Which of the following statements best describes opiates? (A) Opiates will not produce withdrawal. (B) Opiates are not very addictive. (C) Marijuana is an example of an opiate. (D) Opiates are only psychologically addictive. (E) Heroin is an example of an opiate.

e

131. A group of ranchers attempts to discourage coyotes from attacking their sheep by placing a substance on the wool of the sheep that makes coyotes violently ill if they eat it. Very quickly, the coyotes avoid the sheep entirely. In this scenario, what are the UCS, CS, and CR, respectively? (A) Th e substance, the sheep's wool, aversion to the sheep (B) Th e sheep's wool, the substance, aversion to sheep (C) Aversion to sheep, the substance, the sheep's wool (D) Th e coyotes, the sheep's wool, aversion to sheep (E) Th e substance, the sheep's wool, the coyotes

a

132. Th e same ranchers discover that now not only will the coyotes not attack the treated sheep but also they will not attack nearby sheep. Th is is an example of: (A) Extinction (B) Discrimination (C) Generalization (D) Spontaneous recovery (E) Chaining

c

133. In operant conditioning, the Premack Principle states that: (A) Punishment is ineffective. (B) Primary reinforcers are used to reinforce desirable behavior. (C) Punishment is effective when paired with an adversive stimulus. (D) Acquiring a desired behavior from an individual can be eff ectively used as a reinforcer for another, less desirable activity. (E) More desirable behavior can be achieved through positive reinforcement.

d

134. Mrs. Jackson, an English teacher, gives pop quizzes to her students every marking period. Th is is an example of: (A) Variable interval schedule of reinforcement (B) Variable ratio schedule of reinforcement (C) Fixed ratio schedule of reinforcement (D) Fixed interval schedule of reinforcement (E) Interval ratio schedule of reinforcement

a

135. In what manner would Ivan Pavlov have conducted extinction trials on his classically conditioned dogs? (A) Reinforcing the behavior he wished to extinguish (B) Repeatedly presenting the conditioned stimulus (bell) without pairing it with the unconditioned stimulus (food) (C) Repeatedly presenting dogs with the food and the bell at the same time (D) Immediately giving the dogs food (UCS) after the bell (CS) rings (E) Repeatedly bringing in different types of food (UCS) and then reinforcing the salivating immediately after

b

136. In John Watson's "Little Albert" experiment, what was the UCS? (A) Th e white rat (B) Th e little boy (C) Anything white and furry (D) Th e loud noise (E) Fear

d

137. Which of the following is true of classical conditioning? (A) UCS produces UCR (B) CR produces the CS (C) UCR produces the CS (D) CS produces the UCS (E) UCR produces the UCS

a

138. Dylan's mother buys him a sailor's cap before they go on a family fi shing trip. On the boat, Dylan gets nauseated and vomits. Th e next day he gets nauseated just from looking at the sailor's cap. Th e sailor's cap has become: (A) Th e unconditioned stimulus (B) Th e conditioned stimulus (C) Th e conditioned response (D) Th e unconditioned response (E) Th e reconditioned stimulus

b

139. Before Dylan became nauseated, he was able to go fishing with his family, even catching several fish. Fishing is an example of what schedule of reinforcement? (A) Fixed ratio (B) Fixed interval (C) Unfixed interval (D) Variable ratio (E) Variable interval

d

140. Sean sells shoes for a living. His salary depends on how many shoes he can sell in a two-week period of time. What schedule of reinforcement is Sean being paid with? (A) Variable ratio (B) Variable interval (C) Fixed ratio (D) Fixed interval (E) None of the above

c

141. A passenger on an airplane was feeling very anxious about an important job interview the next morning, and as a result he was uneasy and nervous the entire flight. Back home a week later, he is contemplating a holiday trip. Though he hadn't previously been afraid to fl y, he finds himself suddenly nervous about flying and decides to cancel his plans to visit an out-of-state relative. What are the UCS, UCR, CS, and CR, respectively? (A) Job interview, feeling nervous and anxious, flying, feeling nervous and anxious about flying (B) Feeling nervous and anxious, flying, out-of-state relative, feeling anxious and nervous about flying (C) Flying, feeling nervous and anxious, job interview, feeling nervous and anxious (D) Feeling nervous and anxious, job interview, flying, feeling nervous and anxious (E) Job interview, feeling nervous and anxious, out-of-state relative, feeling nervous and anxious

a

142. As part of a new and intriguing line of research in behavioral medicine, researchers gave mice saccharine-flavored water and followed it up with an injection of a drug that weakens mice's immune systems. Later, when these mice drank saccharine-flavored water, they showed signs of weakened immune response. Research is currently under way to see if the reverse is possible (if conditioning can be used to increase immune functioning), a discovery that would surely have important implications for new medical treatments. In this experiment, what is the saccharine-flavored water? (A) Unconditioned stimulus (B) Conditioned stimulus (C) Conditioned response (D) Unconditioned response (E) Stimulus response

b

143. Automobile advertisements, especially those for sports cars, often feature young, beautiful women. Smart advertisers know and research confirms that men rate new cars whose ads include an attractive female as faster, more appealing, better designed, and more desirable than cars with similar ads that do not include an attractive female. What is the unconditioned response? (A) Th e car (B) Th e advertisement (C) Th e attractive women (D) Desire to buy the car (E) Finding the woman attractive

d

144. In the preceding scenario, in terms of classical conditioning, what is the attractive woman? (A) Th e conditioned stimulus (B) Th e unconditioned stimulus (C) Th e conditioned response (D) Th e unconditioned response (E) Th e stimulus response

b

145. Which of the following statements best defines classical conditioning? (A) A type of learning in which behaviors are produced based on rewards and punishments (B) A type of learning based on modeling or imitating the behavior of others (C) A type of learning in which a response naturally elicited by a stimulus comes to be elicited by a formerly neutral stimulus (D) Th e process by which experience or practice results in a change in behavior (E) Th e process by which voluntary behaviors are produced in the presence of certain stimuli

c

146. During the conditioning process of Pavlov's dogs, what element of classical conditioning did the bell and food play? (A) CS and UCS (B) US and CS (C) UCS and CS (D) CS and UCR (E) CS and CR

a

147. Desensitization therapy can best be defined as: (A) A conditioning technique that creates an avoidance of certain foods (B) A conditioning technique that creates a conditioned response from a formerly neutral stimuli (C) A conditioning technique that gradually increases one's desire to perform a particular behavior (D) A conditioning technique that uses generalization to get people to overcome their fears (E) A conditioning technique designed to gradually reduce anxiety about a particular object or situation

e

148. Classical conditioning would best be suited to answer which of the following questions? (A) Why do people repeat behaviors when they are followed by something good? (B) Why do children know a lot about driving a car before their fi rst time behind the wheel? (C) Why do people associate certain foods with nausea? (D) Why are some animals difficult to train to perform certain kinds of behaviors? (E) Why do people imitate behaviors they see someone else get punished for?

c

149. Of the following, which would a psychologist consider the best example of learning? (A) A young man's beard beginning to grow at age 15 (B) A woman experiencing labor pains (C) Salmon swimming upstream during the mating season (D) A child being able to ride a bike (E) A baby sucking on her mother's breast for nourishment

d

150. Th e sight of a needle can trigger fear in some people. Why is this an example of classical conditioning? (A) People learn this when they are young. (B) Th ere is an unconditioned association with fear and the needle. (C) Needles hurt. (D) With positive reinforcement one can overcome their fear. (E) As people get older they overcome this fear.

b

151. What is one major difference between operant conditioning and classical conditioning? (A) Operant conditioning takes place as a result of some voluntary action, while classical conditioning takes place without choice. (B) Operant conditioning takes place before the response, while classical conditioning takes place after the response. (C) Operant conditioning is learned by association, while classical conditioning is learned by reinforcement. (D) Classical conditioning is part of social cognitive learning, while operant conditioning is not. (E) Classical conditioning has a stimulus but no response, while operant conditioning has both a stimulus and a response.

b

152. Suspending a basketball player for committing a flagrant foul is an example of: (A) Negative reinforcement (B) Positive reinforcement (C) Punishment (D) Primary reinforcement (E) Secondary reinforcement

c

153. A defendant is harassed and tortured until he confesses. Th is is an example of: (A) Positive reinforcement (B) Negative reinforcement (C) Punishment (D) Positive punishment (E) Negative punishment

b

154. Punishment can best be defined as: (A) Th e reinforcement of a behavior every time it occurs (B) Taking away something unpleasant when the subject performs the correct behavior (C) An attempt to weaken a response by following it with something unpleasant (D) Adding something unwanted when the subject is not doing the correct behavior and then stopping it when he or she displays the correct behavior (E) Anything that comes to represent a primary reinforcer

c

155. Which of the following statements best explains E. L. Thorndike's law of effect? (A) Behaviors that are negatively reinforced are more likely to discontinue than behaviors that are punished. (B) Receiving reinforcement every time a person performs a good deed, continuous reinforcement, will increase the likelihood that the person will continue that behavior. (C) Th e stimuli of food, water, and sex are innately satisfying and require no learning. (D) Behaviors are strengthened by positive consequences and weakened by negative ones. (E) Behaviors are reinforced through primary reinforcers.

d

156. B. F. Skinner used his "Skinner Box" to work on a procedure in which the experimenter successfully reinforced behaviors, which led up to the desired behavior. Th is procedure is known as: (A) Reinforcement (B) Chaining (C) Primary reinforcers (D) Secondary reinforcers (E) Shaping

e

157. Schedules of reinforcement have a direct effect on maintaining your behavior. Which of the following schedules of reinforcement is identified in this example: Calling a friend and getting a busy signal because he or she is frequently on the phone? (A) Fixed interval (B) Variable interval (C) Fixed ratio (D) Variable ratio (E) Fixed variable

b

158. Which of the following is the best example of a negative reinforcement? (A) A child getting spanked for bad behavior (B) A kindergarten student being put in "time-out" (C) A teenager not being allowed to go to her friend's party (D) A mother taking an aspirin to eliminate her headache (E) A father getting a speeding ticket

d

159. Which of the following best describes the basic principle behind operant conditioning? (A) Th e consequences one receives are directly based on his or her behavior. (B) Th e conditioned stimulus one responds to is called a conditioned response. (C) Continuous reinforcement is the best way to reinforce positive behavior. (D) To decrease undesired behaviors one must use negative punishment. (E) Negative reinforcement and punishment both equally help to rid unwanted behavior.

a

160. What is the goal of both positive and negative reinforcement? (A) To decrease the likelihood that a negative reinforcer will follow a behavior (B) To increase the likelihood that the preceding behavior will be repeated (C) To decrease the likelihood that the preceding behavior will be repeated (D) To ensure there are no negative consequences following the behavior (E) To add a primary reinforcer after someone does a proper behavior

b

161. Latent learning can best be described as: (A) Learning that depends on the mental process (B) Learning that is not immediately reflected in a behavior change (C) A learning technique that provides precise information about one's inner bodily functions (D) Learning that is based on rewards and punishments (E) A type of learning that occurs after the behavior has already been done

b

162. Thorndike's law of effect neglects the inner drives or motives that make learners pursue the "satisfying state," allowing learners to reach their goals. Which of the following psychologists would have agreed with that statement? (A) Kohler (B) Pavlov (C) Tolman (D) Skinner (E) Watson

c

163. Which of the following scenarios is the best example of a cognitive map? (A) A dog sits by the window an hour before her owner should return home. (B) A little girl remembers to get her jacket before leaving for school. (C) A boy follows his big sister home on his bicycle. (D) When asked for directions to his job, a man recites them in great detail. (E) A teacher remembers all the names of her students.

d

164. Wolfgang Kohler conducted a series of experiments in which he placed a chimpanzee in a cage with a banana on the ground just out of his reach outside of the cage. After a period of inaction, the chimp suddenly grabbed the stick in the cage, poked it through the cage, and dragged the banana within reach. Th is type of learning is called: (A) Insight (B) Latent (C) Cognitive (D) Operant (E) Observational

a

165. Harry Harlows's goal was to get his monkeys to figure out that in any set of six trials, the food was always under the same box. Initially the monkeys chose the boxes randomly, sometimes finding food and sometimes not. However, after a while their behavior changed: after two consistent trials of finding the correct box, they continually went back to the same box. Harlow concluded that the monkeys had "learned how to learn." According to Harlow the monkeys established: (A) Cognitive maps (B) Reinforcers (C) Cognitive sets (D) Learned maps (E) Learning sets

e

166. Which of the following statements best exemplifies the idea behind social cognitive learning? (A) Learning occurs when we see someone else being punished for a behavior. (B) Learning is likely to happen whether we see someone else punished or rewarded for behavior. (C) Learning occurs when we see someone else being rewarded for a behavior. (D) Learning is simply based on observation. (E) Learning is based on external rewards and behaviors.

b

167. In Albert Bandura's "bobo" doll experiment, which group of children spontaneously acted aggressively toward the doll rather quickly? (A) Model-reward condition (B) Model-punished condition (C) No-consequences condition (D) Reward and punishment condition (E) No condition

a

168. Devyn watches a violent television show and then pretends to shoot her brother Tyler with a toy pistol. A psychologist would say that Devyn has learned this behavior through: (A) Operant conditioning (B) Classical conditioning (C) Vicarious learning (D) Latent learning (E) Learning set

C

169. Which of the following psychologists would argue that learning can take place when someone is watching another person and performs that behavior even when not reinforced? (A) Edward Tolman (B) Wolfgang Kohler (C) B. F. Skinner (D) John Watson (E) Albert Bandura

E

170. Which of the following responses is not learned through operant conditioning? (A) Shelly gets $50 after getting a 90 percent in her math class. (B) A pigeon learns to peck a disc to get food pellets. (C) A dog learns to turn in circles for a reward. (D) A baby takes his fi rst steps. (E) A horse jumps over a fence to avoid an electric shock.

D

171. Joey is refusing to complete his homework on time. After learning about Joey's love of trains, Mrs. Anderson promises to reward Joey with a Thomas and Friends video upon completion of his next two homework assignments. Th is is an example of: (A) Positive reinforcement (B) Generalization (C) Insight (D) Latent learning (E) The Premack Principle

E

172. While taking his math placement exam, Spencer became stuck on one problem. With only five minutes left, he suddenly arrived at the answer. Th is is an example of: (A) Latent learning (B) Insight (C) Learning set (D) Abstract learning (E) Operant conditioning

B

173. After several attempts at escape with no success, the electrically shocked dogs give up. At that moment the gates open and the dogs could simply walk out, but they don't

D

174. After overcoming her fear of the dentist, Jada finds out she needs a root canal. On her way to the dentist's office, her old fears and anxieties return and she begins to panic. Th is is an example of: (A) Generalization (B) Spontaneous recovery (C) Discrimination (D) Insight (E) Classical conditioning

B

175. Salina receives a one-thousand-dollar bonus at her job after she sold the most cars this month. Th e one-thousand-dollar bonus is an example of a: (A) Primary reinforcer (B) Secondary reinforcer (C) Partial reinforcer (D) Continual reinforcer (E) Total reinforcer

A

176. Katie was able to remember the number 111 by associating it with Admiral Nelson, who happened to have one eye, one arm, and one leg. Th is is an example of: (A) Retrieving (B) Storing (C) Encoding (D) Memory (E) Imagery

C

177. Which of the following examples best illustrates episodic memory? (A) Remembering that you got a bicycle for your 12th birthday (B) Knowing that Christopher Columbus sailed in 1492 (C) Teaching someone how to play tennis (D) Reciting the alphabet (E) Understanding a conversation someone is having in a foreign language

A

178. When asked why she fears spiders, Sophia is unable to explain her fears, where they came from, or how she got them. Th is is an example of: (A) Semantic memory, which helps us avoid painful memories (B) Episodic memory, which has knowledge of specific personal memories (C) Procedural memory, which holds memories that we are not aware of (D) Echoic memory, which holds memories we cannot retrieve (E) Iconic memory, which allows us to forget fear-inducing thoughts

C

179. Which of the following brain structures plays an important role in memory storage, from STM to LTM? (A) Thalamus (B) Hypothalamus (C) Amygdala (D) Hippocampus (E) Cerebrum

D

180. Suppose you are absorbed in reading a novel and a friend asks you a question. You stop reading and ask, "What did you say?" As soon as the words leave your mouth, you realize you can recall your friend's exact words. What is the reason for your ability to play back these words? (A) Iconic memory (B) Echoic memory (C) Semantic memory (D) Sensory memory (E) Short-term memory

B

181. According to the information-processing model, which is the correct order of inputting information? (A) Encode semantically, retrieve elaborately, store information (B) Retrieve from long-term memory, encode in short-term memory, encode in sensory memory (C) Encode in sensory memory, encode in short-term memory, encode in long-term memory (D) Store information, retrieve upon demand, encode necessary information (E) Encode with sensory receptors, store information, retrieve upon demand

E

182. Which of the following statements is not true? (A) Deep processing involves elaborate rehearsal. (B) Automatic processing is unconscious encoding of information. (C) Interference results when new information enters short-term memory and pushes out old information. (D) Levels of processing theory says that remembering depends on how information is encoded. (E) Declarative memory involves memories for skills, habits, and things learned through classical conditioning.

E

183. While walking home from a party drunk, Jeff witnessed a crime. When questioned by the police the following day, he could not remember what he saw. After drinking some liquor, Jeff remembered the crime. Th is phenomenon best illustrates: (A) Th e framing effect (B) Short-term memory loss (C) Hypnotic amnesia (D) State-dependent memory (E) Anterograde amnesia

D

184. Which type of memory is also referred to as working memory? (A) Long-term memory (B) Short-term memory (C) Sensory memory (D) Semantic memory (E) Episodic memory

B

185. Th e ability to maintain exact detailed visual memories over a significant period of time is called: (A) Flashbulb memory (B) Semantic memory (C) Eidetic memory (D) Echoic memory (E) Iconic memory

C

186. The amygdala is responsible for which of the following types of memories? (A) Emotional (B) Procedural (C) Factual (D) Iconic (E) Visual

A

187. Th e primacy effect is best explained by which of the following statements? (A) Items on a list with unique meaning are more likely to be remembered. (B) The first items on a list are likely to be more effectively rehearsed and therefore more likely to be remembered. (C) Items on a list presented more recently are more likely to be remembered. (D) Items on a list with simplistic meaning are more likely to be remembered. (E) Th e last items on a list are more likely to be encoded first and therefore remembered.

B

188. During his English class, Ben is able to recall the author of Th e Scarlet Letter. Th is type of memory is called: (A) Procedural (B) Episodic (C) Long term (D) Semantic (E) Constructive

D

189. Which of the following statements best explains one major difference between short-term memory and long-term memory? (A) Long-term memory is unlimited in capacity while short-term memory is not. (B) Long-term memory holds only episodic memories while short-term memory does not. (C) Long-term memory varies a great deal from one person to another, while short-term memory does not. (D) In terms of processing, long-term memory comes directly after sensory memory while short-term memory does not. (E) Long-term memory depends on neural connections in the limbic system while short-term memory does not.

A

190. Maintenance rehearsal involves: (A) Recalling the words at the end of a list (B) Intentionally repeating information (C) Processing visual memories (D) Systematically recalling information (E) Processing iconic memories

B

191. Linda looks up a telephone number for take-out pizza. She repeats it over and over as she dials the number. However, after giving her order and hanging up, she has forgotten the number. Th is is an example of the use of what memory process? (A) Short-term memory (B) Sensory memory (C) Automatic processing (D) Echoic memory (E) Iconic memory

A

192. After forgetting the combination to several other locks, Nate was trying to find a way to remember the combination to the new lock he bought last week. Th e combination is 19, 20, 9. To remember the combination, he thinks of the year 1929. His method to remember this is an example of: (A) Elaborate rehearsal (B) Maintenance rehearsal (C) Short-term memory (D) Chunking (E) Decoding

D

193. Th e process of encoding information from short-term memory to long term memory is most efficient when it: (A) Has a procedural manner (B) Involves some kind of association (C) Uses repetition (D) Does not use repetition (E) Uses semantic memory

B

194. Maintenance rehearsal is to elaborate rehearsal as: (A) Long-term memory is to short-term memory (B) Sensory memory is to long-term memory (C) Short-term memory is to long-term memory (D) Sensory memory is to short-term memory (E) Automatic memory is to long-term memory

C

195. Which of the following is not an example of effortful encoding? (A) Maintenance rehearsal (B) Repetition (C) Meaningful associations (D) Chunking (E) Transferring information from STM to LTM

D

196. Which of the following statements is correct regarding why eyewitness testimony is not always accurate? (A) People do not have the capacity to remember. (B) People may be asked misleading questions. (C) People do not have a strong recognition. (D) People do not have a strong ability to recall past information. (E) People lie too often.

B

197. What is the correct name of the memory files that contain related information about a specific topic or category? (A) Prototypes (B) Nerve cells (C) Nodes (D) Networks (E) Schemas

C

198. One of the earliest psychologists to study memory and forgetting was Herman Ebbinghaus, who used himself as a subject to test his own recall of a list of nonsense syllables, previously learned through rehearsal. From his work he came up with the concept of a forgetting curve. Th is suggests: (A) Remembering nonsense syllables can be encoded faster than meaningful information. (B) Old information will interfere with new information being encoded into LTM. (C) New information will interfere with old information already stored in LTM. (D) Recall of meaningless information drops very soon after initial learning and then levels off . (E) Recall of meaningless information cannot be retrieved more than three hours after encoding

D

199. Maya is currently enrolled in an Italian class at her local college. While on spring break, Maya travels to Italy. She is excited to practice her new skills, but when she gets there she is having trouble. Every time she tries to speak Italian, Spanish words she learned in high school come out. Th is is an example of: (A) Retroactive interference (B) Proactive interference (C) Retrograde amnesia (D) Anterograde amnesia (E) Dissociative interference

B

200. Retrograde amnesia can best be defined as: (A) Memory loss for events that occur after the time of the incident (B) Memory loss that occurs from childbirth (C) Memory loss for events that have occurred before the time of the incident (D) Memory loss without any specific cause (E) Memory loss for events that have occurred before and after the incident

C

201. Jayden consciously pushes the due date for his term project out of his mind, so much so that on the day it is due, Jayden must take an incomplete from his teacher. Th is is an example of: (A) Repression (B) Aggression (C) Amnesia (D) Forgetting (E) Suppression

E

202. After his car accident, Paul cannot make any new memories. In fact, to remember his daily activities Paul must write everything down. Th is is known as: (A) Retrograde amnesia (B) Anterograde amnesia (C) Proactive interference (D) Retroactive interference (E) Dissociative amnesia

B

203. The method of loci includes which of the following three steps? (A) Create visual places, memorize those places, create vivid imagery (B) Create vivid associations, memorize visual sequences, put associations into places (C) Memorize visual sequence of places, create vivid associations, put associations into selected places (D) Memorize selected places, create vivid imagery, memorize vivid imagery (E) Create vivid associations, memorize associations, put associations into places

C

204. Th e ability to transfer information about words, facts, and events (declarative information) from STM to LTM depends on activity in which part of the brain? (A) Hypothalamus (B) Th alamus (C) Amygdala (D) Hippocampus (E) Medulla

D

205. Talking to yourself over and over again, repeating information silently or out loud, is called: (A) Elaborate rehearsal (B) Rote rehearsal (C) Procedural memory (D) Declarative memory (E) Semantic memory

B

206. Subjects in an experiment learned a sequence of letters (PSQ). Th en they were given a three-digit number (167) and asked to count backwards by threes: 167, 164, 161, and so on, for 18 seconds. At the end they were asked to recall the three letters. Th e subjects showed a rapid decline in their ability to remember the letters. Th is phenomenon is known as: (A) Proactive interference (B) Retroactive interference (C) Decay theory (D) Forgetting curve (E) Episodic interference

C

207. Which of the following exemplifies retrograde interference? (A) Ella failed her French test because she was confusing it with Spanish words she studied last year. (B) Ava, a medical student, failed her test on the bones in the hand because she studied for the bones in the foot after studying the hand. (C) Isabella can no longer form new memories after her head trauma. (D) Nya remembers only the last three items her mom put on the grocery shopping list. (E) Emma cannot remember her third-grade teacher's name, but she does remember her fourth-grade teacher's name.

C

208. After studying for a test, Jack realized he remembered exactly where a particular piece of information appeared on a page in his textbook, even though he did not try to remember the item. Th is is an example of: (A) Explicit memory (B) Procedural memory (C) Declarative memory (D) Implicit memory (E) Semantic memory

D

209. Recognition involves which of the following? (A) Retrieving previously learned information without the presence of any cues (B) Using the available cues to identify information that has already been learned (C) Filling in a specific amount of information without the use of any newly learned cues (D) Using available cues to create an entirely new response (E) Encoding new information to replace previously learned information

D

210. Corey sits at his kitchen table to think about what he needs to buy at the grocery store. He is using his ability to: (A) Recognize (B) Recite (C) Memorize (D) Recall (E) Initiate

D

211. After making a mess of the playroom, Mason visualizes where each toy should be placed in the room. He is using: (A) Method of loci (B) Peg method (C) Visualization (D) Elaborate rehearsal (E) Procedural memory

A

212. Based on Herman Ebbinghaus's forgetting curve research using nonsense syllables, unfamiliar information is: (A) Forgotten within the first eight hours (B) Forgotten within the first hour (C) Forgotten within the first day (D) Forgotten within the first two days (E) Forgotten within the first week

B

213. Th e forgetting curve measures which of the following? (A) Th e amount of previously learned information that subjects remember across time (B) Th e amount of new information that can remain in the short-term memory (C) Memory that cannot be consciously remembered over time (D) Th e amount of information children can retain over age five (E) Th e amount of information one can memorize in any given day

A

214. Recognition is to recall as: (A) Fill-in is to multiple choice (B) Fill-in is to essay (C) Multiple choice is to fill-in (D) Multiple choice is to essay (E) Multiple choice is to matching

C

215. Kimberly knows she did something embarrassing at her friend's birthday party many years before, but she cannot remember what it was. Th is is an example of: (A) Repression (B) Amnesia (C) Forgetting curve (D) Implicit memory (E) Interference

A

216. Mental age can best be defined as: (A) A method of estimating a child's intellectual ability by comparing the child's score on intelligence tests and his or her age (B) A method of estimating a child's intellectual ability based on raw scores on intelligence tests (C) Comparing a child's actual age with his or her computed age (D) Basing a child's age level on his or her scores on a standardized test (E) Charting a child's age based on the level of correct responses on an intelligence test

A

217. When an intelligence test measures what it is supposed to, the test is considered to be: (A) Reliable (B) Valid (C) Accurate (D) Standardized (E) Comparative

BB

218. Which of the following psychologists believed that intelligence was a collection of mental abilities? (A) Wechsler (B) Broca (C) Binet (D) Terman (E) Galton

C

219. Th e extent to which traits, abilities, or IQ scores may increase or decrease as a result of environmental factors is called: (A) Nature-nurture question (B) Heritability (C) Independent variables (D) Reaction range (E) Ecological testing

D

220. If a four-year-old girl correctly answered questions on an intelligence exam similar to a five-year-old girl, she would be said to have a mental age of five. In this case her intelligence quotient (IQ) would be: (A) 100 (B) 95 (C) 150 (D) 125 (E) 110

D

221. Which of the following psychologists added a performance scale in an attempt to measure nonverbal skills and rule out other cultural or educational biases? (A) Wechsler (B) Binet (C) Gardner (D) Sternberg (E) Terman

A

222. Charles Spearman's two-factor theory of intelligence referred to which of the following? (A) Mathematical skills and spatial intelligence (B) Analytical problem solving and interpersonal skills (C) Ability to perform complex mental work and mathematical or verbal skills (D) Analytical problem solving and intrapersonal skills (E) Ability to reason logically and demonstrate written language and thinking skills

C

223. Robert Sternberg's triarchic theory of intelligence was divided into three categories. Which three categories are correct? (A) Practical, intrapersonal, creative (B) Experimental, analytical, problem solving (C) Experimental, problem solving, practical (D) Analytical, logical, practical (E) Analytical, problem solving, practical

E

224. According to Raymond Cattell, what is the major difference between crystallized intelligence and fluid intelligence? (A) Crystallized intelligence refers to problem-solving abilities, while fluid intelligence is the ability to absorb and retain information. (B) Crystallized intelligence is the ability to absorb and retain information, while fluid intelligence refers to problem-solving abilities. (C) Crystallized intelligence is the ability to be analytical, while fluid intelligence is the ability to read and write. (D) Crystallized intelligence is the ability to read and write, while fluid intelligence is the ability to be analytical. (E) Crystallized intelligence is the ability to absorb information, while fluid intelligence is the ability to analyze the information.

B

225. An architect would likely have good spatial intelligence, a gymnast or dancer would likely have good body-kinesthetic intelligence, and a psychologist would probably have good intrapersonal skills. Which of the following psychologists would agree with this statement? (A) Gardner (B) Spearman (C) Th urstone (D) Guilford (E) Terman

A

226. On a normal distribution of IQ test scores, with a mean of 100 and a standard deviation of 15 points, a score of 85 places you approximately in what percentile of the population? (A) 16th (B) 50th (C) 97th (D) 76th (E) 24th

A

227. Which of the following types of tests measures the capacity of a test taker to perform some task or role in the future? (A) Achievement (B) Aptitude (C) Conventional (D) Self-monitored (E) Adult intelligence scale

B

228. A savant can best be defined as: (A) A mentally handicapped individual (B) A child between the ages of 8 and 12 who suffers from autism (C) A mentally handicapped individual with exceptional ability in mathematical calculations, memory, art, or music (D) A male adult who suffers from delusional thoughts and erratic behavior (E) A social loner who has exceptional abilities in the field of science or math

C

229. Which of the following psychologists did not suggest the existence of more than one kind of intelligence? (A) Gardner (B) Sternberg (C) Guilford (D) Th urstone (E) Spearman

E

239. In a normal distribution of IQ scores, what percentage of people has a score between 85 and 115? (A) 35 percent (B) 54 percent (C) 68.26 percent (D) 79.32 percent (E) 95.44 percent

C

240. A normal distribution is one in which: (A) Th e majority of scores are high. (B) Th e majority of scores are low. (C) All scores fall in the middle range. (D) Th e majority of scores fall in the middle range. (E) All scores are above the mean.

D

241. Because it has all the features commonly associated with the concept of a dog, a poodle is considered: (A) An algorithm (B) A heuristic (C) A prototype (D) A phoneme (E) A concept

C

242. Which of the following is an example of functional fixedness? (A) Using a blanket as a floor mat (B) Not being able to solve a math problem because you are using the incorrect formula (C) Replacing oil with applesauce when baking a cake (D) Failing to use your keys to open a package when you can't find a pair of scissors (E) Picking up a tangerine and calling it an orange

D

243. To become a chess or checkers champion one must use: (A) Algorithms (B) Heuristics (C) Concepts (D) Prototypes (E) Morphemes

A

246. On her way to London, Janet was invited into the cockpit to meet the pilot, Alex. She was surprised to see that Alex was a woman. Th is is an example of: (A) Confirmation bias (B) Convergent thinking (C) Insight (D) Representative heuristic (E) Availability heuristic

D

247. Phonemes are best defined as: (A) The smallest meaningful combination of sounds in a language (B) Th e basic sounds of consonants and vowels (C) Something that specifies the meaning of words and phrases (D) A set of rules that specify how we combine words to form meaningful sentences (E) A special form of communication

B

248. Noam Chomsky's language theory included the idea that: (A) Language development occurs between the ages of three and five. (B) Children learn language through positive and negative reinforcement. (C) Children make the same grammatical errors as their parents. (D) Children model language development from those around them. (E) Children have an innate mental grammar.

E

249. There is evidence to support the idea that there is an inborn tendency to absorb language. Which of the following psychologists would agree with this statement? (A) Chomsky (B) Whorf (C) Skinner (D) Saffron (E) Sapir

A

250. Which of the following statements is not supported by the Whorf-Sapir linguistic relativity hypothesis? (A) Th e language a person speaks determines the way a person thinks. (B) If language lacks expression, the thought that corresponds will likely not occur. (C) Th ere is evidence to support that language development has inborn tendencies. (D) If language affects our ability to store information, it should affect our thought process. (E) To understand new vocabulary, it is easier to think about the relationship between language and thought.

C

251. Suppose you consider elderly people to be infirm and mentally slow. Every time you see elderly people in need of care or assistance, you take it as evidence of your belief, while ignoring the many cases of healthy, active elderly people. Th is is an example of: (A) Representative heuristic (B) Availability heuristic (C) Prototype (D) Confirmation bias (E) Functional fixedness

D

252. Angie and Brad are looking to buy a new home. One criterion is a preference for a brick house. However, they would consider changing their minds and buying a wood house if it were located in a good school district and reasonably priced. In this case the attractive features off set the lack of brick exterior. Th is is an example of: (A) Representative heuristic (B) Compensatory model (C) Noncompensatory model (D) Availability heuristic (E) Confirmation bias

B

253. Which of the following sentences best explains the idea of over regularization? (A) Yesterday I goed to the store. (B) I ain't going to the store. (C) I no want to go to store. (D) I want store. (E) No store please.

B

256. Which of the following statements best illustrates the concept of framing? (A) A PSA for breast mammograms chooses to use the statement "you can die if you don't," rather than "this can save your life." (B) Lily assumes her doctor named Chris is a male, when in fact she is female. (C) An advertiser uses divergent thinking to come up with a commercial slogan. (D) A person remembers items on a list depending on which order they appear in. (E) A cigarette company puts beautiful women in its commercials.

A

257. Which of the following terms is an example of an innate sound program in the brain that involves making and processing sounds that will eventually be used to form words? (A) Grammar (B) Babbling (C) Talking (D) Sentences (E) Morphemes

B

258. Rules of grammar can best be defined as: (A) Acquiring language through four stages (B) Phonemes and morphemes (C) Problem solving using language (D) Forming sentences that range from three to eight words (E) Speaking in sentences that are stated in different ways but have the same meaning

A

259. Which of the following is not a good example of the ability to overcome functional fixedness? (A) A potato is used as a temporary gas cap. (B) A paper clip is used to make earrings. (C) A glass is used as a paperweight. (D) A credit card is used as a bookmark. (E) A math formula is used to solve a math problem

E

260. To develop a concept of an office, the definition theory states that one must: (A) List all essential features of an office (B) Construct an ideal office (C) Look at the average office (D) Visit various offices (E) Transform a room into an office

A

261. Which of the following statements best describes an example of availability heuristic? (A) After speaking in front of 200 people, Tim is no longer afraid of public speaking. (B) Jane thinks all men will eventually cheat on her. (C) Steven complains to his wife about work after a very bad day, but at the office party Steven's wife sees how much he enjoys what he does. (D) Rob claims that when he is confronted with a problem, he likes to come up with one correct solution. (E) After meeting a celebrity, Todd now wants to become an actor and eventually become famous.

C

262. Which of the following statements best defines information retrieval? (A) Having memories of your 16th birthday party (B) Th inking all dog owners are sensitive people (C) Memorizing information that might be needed in an emergency (D) Picking out the proper outfit to wear to a friend's housewarming party (E) Writing a term paper

C

263. When solving an anagram one must try every possible combination of letters until the hidden word appears. This is an example of: (A) A heuristic (B) A concept (C) A subgoal (D) An image (E) An algorithm

E

264. Which of the following terms is not an example of a problem-solving technique? (A) Functional fixedness (B) Trial and error (C) Subgoals (D) Brainstorming (E) Heuristics

A

265. This problem-solving technique involves analyzing the difference between the current situation and the desired end, and then doing something to reduce that difference. (A) Subgoals (B) Means-end analysis (C) Brainstorming (D) Heuristic (E) Algorithm

B

266. Motivation can best be defined as: (A) An innate biological force that produces a fixed set of behaviors (B) Various physiological and psychological factors that cause a person to act in a particular way (C) A biological state in which an organism lacks something essential for survival (D) The tendency or need for a body to stay in a balanced state (E) Environmental factors that reward, reinforce, or encourage our behavior

B

267. Repulsion, curiosity, pugnacity, and humility are all examples of (A) Needs (B) Emotions (C) Instincts (D) Motivations (E) Incentives

C

268. If a person does not eat for a period of time, it causes a need for food. Th is need produces a state of tension. Th e tension energizes the person to act in some way to find food, thereby returning the body to homeostasis. This is an example of: (A) Intrinsic action pattern (B) Sympathetic nervous system (C) Extrinsic motivation (D) Drive reduction theory (E) Biological needs

D

269. A fixed action pattern is best illustrated by which of the following examples? (A) Jackie's need to climb mountains (B) Marlon's motivation to make a lot of money (C) A baboon rising on hind feet when threatened (D) A dog sitting by the window an hour before his owner comes home (E) Michael's cat purring when she hears the can opener

C

270. Which of the following examples best illustrates an intrinsic motivation? (A) Running a marathon to support breast cancer (B) Rock climbing to win first prize (C) Graduating with honors (D) Trying out for the high school basketball team (E) A teacher praising a student when she raises her hand

A

271. According to Maslow's hierarchy of needs, an adolescent who is beginning to form serious romantic relationships would be in what level? (A) Level 1 (B) Level 2 (C) Level 3 (D) Level 4 (E) Level 5

C

272. According to Maslow's hierarchy of needs, an individual who quits his job and moves to Africa to do philanthropic work would be in what level? (A) Level 1 (B) Level 2 (C) Level 3 (D) Level 4 (E) Level 5

273. Which of the following statements best illustrates Maslow's esteem needs? (A) Moving to a safe community to raise your children (B) Going to school to earn a master's degree in counseling (C) Getting married to your high school sweetheart (D) Donating a large sum of money to charity (E) Going to the gym three days a week to improve your health

E

274. Which of the following factors signals hunger in our body? (A) High levels of glucose (B) Stimulation of the lateral hypothalamus (C) Stimulation of the ventromedial hypothalamus (D) High levels of cholecystokinin (E) Stomach contractions

B

275. Homeostasis is best defined as: (A) Th e physiological need to satisfy your hunger or thirst (B) Th e body's tendency to maintain balance (C) Th e arousal of the autonomic nervous system (D) Th e release of the hormone serotonin (E) Th e biological need for safety and security

B

276. If the ventromedial hypothalamus of a rat is destroyed: (A) Th e rat will starve to death. (B) Th e rat will only eat when it feels hungry. (C) Th e rat will begin to feel full. (D) Th e rat will become obese. (E) Th e rat's blood glucose level will remain constant.

D

277. Which of the following statements best defines set point? (A) It refers to how efficiently the body breaks food down. (B) It refers to how quickly the body turns food into energy. (C) It controls the body's metabolism. (D) It plays a role in influencing appetite. (E) It refers to a certain level of body fat that the body maintains.

E

278. An individual with a low metabolic rate is: (A) More likely to have a fatter body (B) Less likely to have a fatter body (C) Less likely to store excess fuel (D) More likely to eat more than someone with a high metabolic rate (E) More likely to have an easier time losing weight 279. Which of the following is not an example of a psychological hunger factor? (A) Social-cultural (B) Learned associations (C) Personality traits (D) Peer pressure (E) Nutrition

A

280. An individual's subjective experience and feeling of being either a male or female is referred to as: (A) Gender roles (B) Sexual orientation (C) Gender identity (D) Transgender (E) Sex categories

C

281. By age five, children have acquired many of the complex thoughts and behaviors that accompany being male or female. Th is is best known as: (A) Gender identity (B) Gender roles (C) Sexual identity (D) Sexual cognition (E) Gender cognition

B

282. Which of the following brain structures is most responsible for hunger and satiety, respectively? (A) The ventromedial hypothalamus, the lateral hypothalamus (B) The lateral hypothalamus, the ventromedial hypothalamus (C) The amygdala, the hippocampus (D) The hippocampus, the amygdala (E) The cerebellum, the lateral hypothalamus

B

283. Which of the following statements is the core concept of Maslow's hierarchy of needs? (A) Individuals who fail to reach self-actualization feel a sense of failure. (B) Level 1 is the need for safety and security. (C) Men are more concerned with safety needs and women are more concerned with esteem needs. (D) Physiological needs must be met before an individual can attain self actualization. (E) An individual can skip levels 1, 2, and 3 and go directly to finding success at level 4.

D

284. A journalist chooses to go to Afghanistan to cover the war hoping to acquire the admiration of his peers and a promotion. Which of the following theories of motivation best explains this decision? (A) Drive theory (B) Incentive theory (C) Fixed action pattern (D) Socio-cognitive theory (E) Motivation

B

285. Motivation starts with an individual's: (A) Emotion (B) Arousal (C) Need (D) Drive (E) Incentive

C

286. Which of the following sequences is correct according to the James-Lange theory of emotion? (A) Physiological changes, feel emotion, interpretation of emotion, observable behavior (B) Physiological changes, interpretation of physiological change, feel emotion, observable behavior (C) Feel emotion, physiological changes, interpretation of physiological change, observable behavior (D) Brain interpretation, physiological changes, observable behavior (E) Interpretation of stimuli, brain interpretation, physiological changes, observable behavior

B

287. What was the name of the theory on emotion that originated from the work of Charles Darwin? (A) Facial feedback theory (B) Cannon-Bard theory (C) Cognitive-appraisal theory (D) Affective-primacy theory (E) Two-factor theory

A

288. What was one major criticism of the James-Lange theory on emotion? (A) Emotions are usually associated with one specific physiological change in the body. (B) Physiological changes do not vary in intensity. (C) Different emotions are not necessarily associated with different patterns of physiological responses. (D) Most emotions do not need a large amount of interpretation. (E) Cognition has no direct affect on the physiological changes in the body.

C

289. Which of the following theories on emotion assumes that our interpretation or appraisal of a situation is the primary cause of emotion? (A) Cannon-Bard theory (B) Facial feedback theory (C) James-Lange theory (D) Schachter-Singer theory (E) Peripheral theory

D

290. Which of the following statements best supports the Schachter-Singer theory of emotion? (A) A friend walks up to you and tells you he saw someone back into your car and drive away—making you angry. (B) You hear a loud noise, your heart starts to pound, and you know you are scared. (C) You feel sad because you are crying. (D) You know you are happy because you have been smiling all day. (E) Your heart is racing but you are not sure why.

A

291. Unlike the cognitive-appraisal theory, the affective-primacy theory states: (A) Physiological changes in the body happen simultaneously with the brain's interpretation of an event. (B) Th e brain is entirely responsible for interpretations of any emotion one is having. (C) Physiological changes in the body often determine the emotion one is feeling. (D) In some situations, a person feels an emotion before having time to interpret the situation. (E) Sometimes a person's interpretation of a situation is the primary cause of an emotion.

D

292. Th e six universal emotions specified that inherited facial patterns of expression are: (A) Worried, sadness, anger, resentment, disgust, fear (B) Happiness, sadness, anger, surprise, disgust, fear (C) Happiness, excitement, anger, sadness, fear (D) Confusion, happiness, sadness, anger, fear, disgust (E) Happiness, sadness, resentment, anger, disgust, fear

B

293. Which of the following statements supports the results of the Ekman Friesen experiment? (A) Between five and seven months of age, infants show fear. (B) It is difficult to communicate with people of different cultures. (C) People show disgust for many different reasons. (D) Babies smile due to modeling behavior. (E) Children exhibit emotion differently than adults do.

A

294. Which statement best exemplifies the Yerkes-Dodson law? (A) Many of Leo's friends think he is depressed due to his lack of any facial expressions. (B) Josh has a look of disgust on his face after smelling the rotten milk. (C) Linda spends hours playing a challenging video game because this activity arouses and motivates her. (D) William's test anxiety helps him score higher on the exam. (E) Jacob falls asleep in his physics class after being so bored.

D

295. Happiness includes all of the following except: (A) Feeling a positive emotion (B) Being satisfied with your life (C) Not experiencing a negative emotion (D) Having a high-paying job (E) Both environmental and inherited factors

D

296. Three weeks after winning the lottery, Tanya gave birth to Sophia. Tanya claimed that winning the lottery was the most exciting thing to happen to her until she gave birth to her daughter. Th is is an example of: (A) Th e adaptation level theory (B) Th e rules of happiness theory (C) Th e Yerkes-Dodson law (D) Th e psycho-revolutionary theory (E) Th e relative deprivation theory

A

297. Which statement best defines display rules? (A) Specific inherited facial patterns or expressions are universal. (B) People innately have a tendency to show affection in public. (C) Many cultures do not allow public displays of emotion. (D) Specific cultural norms regulate how much emotion we express socially. (E) In some situations people feel an emotion before they have time to appraise the situation.

D

298. An interpretation or appraisal of a situation as having a positive or negative impact on your life resulting in a subjective feeling is called: (A) The affective-primacy theory (B) Th e James-Lange theory (C) Th e Cannon-Bard theory (D) Th e facial feedback theory (E) Th e cognitive-appraisal theory

E

299. While sitting in a waiting room, a man next to you begins yelling and acting aggressively. Your heart begins beating fast. You interpret your environmental cues as the cause of your arousal. Which theory of emotion would support this explanation? (A) Th e James-Lange theory (B) Th e Schachter-Singer theory (C) The affective-primacy theory (D) All of the above (E) None of the above

B

300. Which of the following theories of emotion support the idea that emotions and bodily responses occur simultaneously? (A) The James-Lange theory (B) The Schachter-Singer theory (C) The Cannon-Bard theory (D) The cognitive-appraisal theory (E) The affective-primacy theory

C

301. Which of the following theories of emotion supports the importance of an individual's personal assessment of a situation? (A) Th e cognitive-appraisal theory (B) Th e Cannon-Bard theory (C) Th e facial feedback theory (D) Th e James-Lange theory (E) Th e peripheral theory

A

302. "We feel sorry when we cry and afraid because we tremble." Th is quote is supported by which theory of emotion? (A) Th e Cannon-Bard theory (B) Th e James-Lange theory (C) Th e Cannon-Lange theory (D) Th e James-Bard theory (E) Th e facial feedback theory 303. Larry really wants to buy his wife the diamond watch she always wanted for her birthday, but he knows he should be more conservative with his money. What type of conflict is he facing? (A) Approach-approach (B) Approach-avoidance (C) Avoidance-avoidance (D) Positive approach (E) Negative approach

B

304. Which of the following examples best illustrates the concept of approach-approach conflict? (A) Ariel must work at Bloomingdale's while in college. (B) Sabrina is forced to call the home of one of her students because he is not doing his homework. (C) Latoya has to choose between Princeton and Yale University. (D) Wendy just got a promotion, but she now has to fire someone else. (E) Randy works as a stand-up comedian, but he needs to make more money.

C

305. According to the Cannon-Bard theory of emotion, which part of the brain is vital in terms of physiological responses to emotion? (A) The cerebellum (B) The temporal lobe (C) The frontal lobe (D) The limbic system (E) The left hemisphere

D

306. If we are about to jump out of an airplane for the first time, we tend to feel extreme fear along with low levels of elation. Later, when we decide to jump again, we experience more elation and less fear. This scenario is supported by which theory of emotion? (A) The James-Lange theory (B) The affective-primacy theory (C) The opponent-process theory (D) The Cannon-Bard theory (E) None of the above

C

307. Emotional responses develop before complex thinking occurs. Which of the following psychologists would agree with this statement? (A) Ekman (B) Schachter (C) Bard (D) Lange (E) Zajonc

E

308. Which area of the brain is extremely stimulated when an individual is feeling sad? (A) Hypothalamus (B) Thalamus (C) Temporal lobe (D) Parietal lobe (E) Amygdala

A

309. Which example best illustrates the adaptation level theory? (A) Michelle takes her mother's inheritance for granted. (B) Regina has so much to do with so little free time to do it in. (C) Natasha hired another assistant to help lessen her workload. (D) Cathryn lost her brand-new wallet and bought a more expensive one. (E) Alexis sold her engagement ring to send her son to college.

A

310. To achieve high performance on a simple task, the Yerkes-Dodson law recommends: (A) High arousal (B) Low arousal (C) Medium arousal (D) Extreme anxiety (E) Moderate anxiety

A

311. Cross-sectional research differs from longitudinal research in that: (A) Cross-sectional research studies the developmental changes of subjects who are of different ages. (B) Cross-sectional research studies developmental changes using the same group of subjects over time as they grow older. (C) Cross-sectional research is more reliable than developmental research. (D) Cross-sectional research is too specific to the group of people being used for research. (E) Cross-sectional research takes too much time to gather results.

A

315. Research has determined that, between the ages of 6 and 12 months, all babies have acquired: (A) Three-dimensional dreaming (B) Th e ability to walk (C) Th e ability to talk (D) Depth perception (E) Potty-training abilities

D

316. Motor development in babies develops in a proximodistal fashion. This is best described as: (A) From nearest to the center of the body to the farthest from the center (B) From the top of the head to the bottom of the feet (C) From the farthest from the center to the nearest to the center (D) From the bottom of the body to the top of the body (E) From the left of the body to the right of the body

A

317. Which of the following statements best defines maturation? (A) It is directly based on social cognitive learning. (B) It is the basis for all physiological and psychological development. (C) It is an automatic biological development of the body and nervous system that naturally unfolds over time. (D) It does not take place in all human beings. (E) It is directly associated with genetic links.

C

319. Jean Piaget defined egocentrism as: (A) The belief that young adults don't listen to their parents (B) The idea that preschool children cannot see things from another's point of view (C) The understanding that young children cannot learn outside of a structured classroom (D) The idea that young children are selfish and grow out of it over time (E) The belief that children cannot do more than one task at a time

B

320. According to Jean Piaget, what type of learning do individuals acquire during the formal operational stage? (A) Abstract thought (B) Symbolism (C) Memorization skills (D) Visual learning (E) Auditory learning

A

321. An awareness that objects continue to exist when out of sight is called: (A) Mental images (B) Sensory-motor (C) Object permanence (D) Object understanding (E) Conservation

C

322. According to Jean Piaget, children understand the concept of symbolism during which stage of development? (A) Sensory-motor (B) Preoperational (C) Concrete operational (D) Formal operational (E) Operational

B

323. One major difference between assimilation and accommodation is that assimilation: (A) Is a process by which children use old methods to deal with new situations (B) Is a process by which children change their thought process to meet the needs of their world (C) Is a process by which children gain an understanding of the world around them (D) Is a process by which individuals shape their lives based on learned observations (E) Is a process by which individuals begin using hypothetical thinking skills

A

324. According to Lawrence Kohlberg, during the preconventional stage of moral development children tend to: (A) Use abstract thoughts or principles to determine their behavior (B) Make behavioral decisions based on legal issues (C) Understand morality based on customs or values (D) Interpret behavior in terms of concrete consequences (E) Define good behavior as that which pleases other people

D

325. During a discussion in class regarding cheating in school, a student argues, "Cheating is wrongit is important to follow rules." Lawrence Kohlberg would say this student is in what stage of moral development? (A) Preconventional (B) Conventional (C) Postconventional (D) Nonconventional (E) Advanced conventional

B

326. As a preschooler, Emma has developed a number of cognitive and social skills that she will use to assume responsibility. According to Erik Erikson, what stage of psychosocial development is Emma in? (A) Trust versus mistrust (B) Autonomy versus self-doubt (C) Initiative versus guilt (D) Industry versus inferiority (E) Identity versus role confusion

C

327. When Daniel begins walking, talking, and exploring, he is bound to get into conflict with his parents. If his parents punish his explorations, Daniel may develop a feeling that independence is bad. According to Erik Erikson, what stage of psychosocial development would this occur in? (A) Identity versus role confusion (B) Industry versus inferiority (C) Initiative versus guilt (D) Autonomy versus self-doubt (E) Trust versus mistrust

D

328. According to Sigmund Freud, what is the correct order of the five psychosexual stages of development? (A) Oral, anal, phallic, early, genital (B) Oral, anal, phallic, latency, genital (C) Anal, phallic, latency, genital, oral (D) Genital, oral, latency, phallic, anal (E) Phallic, anal, oral, latency, genital

B

329. If individuals successfully solve their problems during each stage of life, they will develop good social traits. If they do not, their problem-solving skills will be hindered, causing new problems at the next stage. Which psychologist(s) would agree with this statement? (A) Freud (B) Piaget (C) Erikson (D) A and B (E) A and C

E

330. Monica is extremely neat and orderly. She cannot stand it when people touch things on her desk. She also has a problem lending money to even her closest friends. Freud would say she is stuck in what psychosexual stage? (A) Oral (B) Anal (C) Phallic (D) Latency (E) Genital

B

331. Jenna is extremely sarcastic. She uses this to cover up her low self-esteem. Freud would say she is stuck in which psychosexual stage? (A) Oral (B) Anal (C) Phallic (D) Latency (E) Genital

A

332. If a child believes stealing in order to save a life is OK because life is even more important than following the law, this child would be in what stage of moral development? (A) Level one: preconventional (B) Level two: conventional (C) Level three: postconventional (D) Level four: operational (E) Level five: formal operational

C

333. Which of the following statements is a major criticism of Jean Piaget's work with cognitive development? (A) Piaget failed to include clear age differences for his stages. (B) Piaget placed too much emphasis on cognitive differences between young children and adolescents. (C) Piaget often overestimated the cognitive abilities of children. (D) Piaget often underestimated the cognitive abilities of children. (E) Piaget gave little credit to other psychologists who helped him develop his theory.

D

334. Th e rooting reflex is an infant's tendency to: (A) Th row legs up in the air (B) Wave arms when startled (C) Open mouth and turn head when touched on the cheek (D) Follow a moving object with eyes (E) Grasp nearby objects

C

335. Harry Harlow's experiment with monkeys and surrogate mothers emphasized the importance of: (A) Satisfying hunger (B) Body temperature (C) Fulfilling needs (D) Intrinsic motivation (E) Contact

C

336. When adolescents were asked about their major concerns for their future, top answers on their lists were getting married, having friends, getting a job, and doing well in school. Each of these concerns involves the ability to understand abstract thought and concepts. According to Jean Piaget, what stage of cognitive learning is this? (A) Sensorimotor (B) Preoperational (C) Operational (D) Formal operational (E) Postoperational 337. While at her friend's party, Angelica begins to feel self-conscious because she assumes everyone is staring at her. According to David Elkind, what adolescent belief is Angelica feeling? (A) Imaginary audience (B) Personal fable (C) Awkwardness (D) Insecure attachment (E) Imaginary person

338. Which of the following examples best illustrates a personal fable? (A) Talia feels insecure when she is making her presentation in front of her class. (B) Lola feels as though no one else could possibly be so much in love as she is. (C) Dina lies to her parents about going to her boyfriend's birthday party. (D) Alexa is no longer interested in her schoolwork

all she wants to do is hang out with her friends. (E) Joanna falls into a severe depression when her boyfriend breaks up with her.

339. Authoritative parents can best be defined as: (A) Parents who befriend their children and do not use discipline (B) Parents who are less controlling and behave with a more accepting attitude (C) Parents who try to control the behavior of their children in accordance with a set standard of conduct (D) Supportive parents who discuss their rules and policies with their children (E) Parents who command obedience and teach their values with little communication

340. According to Erik Erikson, what is one major conflict teenagers deal with during the identity versus role confusion stage of psychosocial development? (A) Finding a more purposeful life as an adult (B) Achieving personal satisfaction (C) Reflecting on previous life challenges (D) Finding intimacy by developing loving relationships (E) Achieving generativity through family relationships

341. According to Robert Sternberg, what are the three components of love? (A) Passion, romance, attraction (B) Commitment, intimacy, companionship (C) Passion, intimacy, commitment (D) Intimacy, trust, attraction (E) Intimacy, companionship, attraction

342. Carol Gilligan believed moral decision making is dependent primarily on which of the following? (A) Age (B) Culture (C) Religion (D) Gender (E) Nationality

343. Which of the following parenting styles results in the most socially responsible adults? (A) Authoritarian (B) Authoritative (C) Permissive (D) Autocratic (E) Sensitive

344. A failure to develop a consistent identity results in: (A) Role confusion (B) Inferiority (C) Insecurity (D) Stagnation (E) Social isolation

345. According to Erikson, teachers, friends, and other people outside of the home first become important in shaping attitudes of a child during what psychosocial stage? (A) Autonomy versus self-doubt (B) Initiative versus guilt (C) Industry versus inferiority (D) Integrity versus despair (E) Trust versus mistrust

346. Daniel Levinson studied: (A) Child development (B) Adolescent behavior (C) Death and dying (D) Male adult psychosocial stages (E) Female adult psychosocial stages

347. According to Erikson, a child who is learning the importance of academic success in school based on receiving a report card is in what psychosocial stage? (A) Industry versus inferiority (B) Generativity versus stagnation (C) Identity versus role confusion (D) Initiative versus guilt (E) Integrity versus despair

348. I am in my early fifties. If I do not reach out to others, especially young people, Erik Erikson says I will experience: (A) Shame (B) Depression (C) Isolation (D) Stagnation (E) Despair

349. According to Lawrence Kohlberg, behavior directed by self-accepted moral principles is an example of what stage of moral development? (A) Preconventional (B) Conventional (C) Postconventional (D) Nonconventional (E) Unconventional

350. According to Erik Erikson, as a young adult you are most interested in developing: (A) Initiative (B) Integrity (C) Generativity (D) Trust (E) Intimacy

351. Which group of stages from Erikson, Kohlberg, and Levinson identify the same phase of life? (A) Identity, conventional, age 50 crisis (B) Generativity, postconventional, age 50 crisis (C) Generativity, preconventional, midlife transition (D) Intimacy, preconventional, midlife transition (E) Initiative, conventional, age 30 crisis

352. According to Freud adolescents are in what psychosexual stage? (A) Oral (B) Anal (C) Phallic (D) Latency (E) Genital

353. Which three psychologists focused their work on adolescent development? (A) Freud, Kohlberg, Gilligan (B) Gilligan, Erikson, Havighurst (C) Havighurst, Elkind, Marcia (D) Marcia, Levinson, Elkind (E) Elkind, Freud, Piaget

354. As children begin their elementary school years, they enter Erikson's stage of: (A) Trust versus mistrust (B) Autonomy versus doubt (C) Initiative versus guilt (D) Industry versus inferiority (E) Identity versus role confusion

355. Robert Havighurst believed adolescents must: (A) Complete a series of tasks (B) Fall in love (C) Graduate college (D) Get along with their parents (E) Find a summer job

356. Claire just celebrated her 90th birthday with her family and close friends. According to Erik Erikson, she has probably achieved: (A) Isolation (B) Integrity (C) Despair (D) Autonomy (E) Stagnation

357. Which is the correct order of the five stages of dealing with death or loss? (A) Denial, anger, bargaining, depression, acceptance (B) Anger, denial, bargaining, depression, acceptance (C) Bargaining, anger, denial, depression, acceptance (D) Depression, bargaining, anger, denial, acceptance (E) Depression, anger, denial, bargaining, acceptance

358. Which of the following psychologists formulated a stage theory addressing our encounters with grief? (A) Sigmund Freud (B) Erik Erikson (C) Elisabeth Kubler-Ross (D) Carol Gilligan (E) Lawrence Kohlberg

359. In late adulthood, individuals experience a decrease in which of the following? (A) Sexual desire (B) Cognitive abilities (C) Creativity (D) Intellect (E) Compassion for others

360. Ethel, who is 80 years old, lost her husband last year, and her children hardly ever come to visit. She looks back on her life with a lot of regret. According to Erik Erikson she is experiencing: (A) Stagnation (B) Depression (C) Regression (D) Despair (E) Isolation

361. Freud's psychodynamic theory of personality emphasizes: (A) The importance of early childhood experiences (B) The importance of sibling rivalries (C) The role genetics plays in personality development (D) The nature-nurture debate (E) The conscious thought process only

362. To explain why we do things that we cannot explain, Freud used the concept of: (A) Conscious forces (B) Subconscious tendencies (C) Unconscious motivation (D) Preconscious motivation (E) Conscious association

363. Th e Freudian technique in which clients are encouraged to talk about any thoughts that enter their mind to help with uncensored talk is called: (A) Unconscious motivation (B) Free association (C) Free analysis (D) Freudian interpretation (E) Psychodynamic theory

364. Freud believed the mental process must have a source of energy called: (A) Ego (B) Superego (C) Id (D) Conscious (E) Unconscious

365. As children learn they must follow rules and regulations in satisfying their wishes, they develop: (A) A superego (B) An id (C) An ego (D) A preconscious (E) An unconscious

366. As infants discover that parents put restrictions on satisfying their wishes, infants learn to control their wishes. According to Freud they do this through the development of: (A) An id (B) A superego (C) An ego (D) A conscious (E) A subconscious

367. Which of the following examples best illustrates the pleasure principle? (A) A student takes pleasure in reporting a fellow classmate for cheating. (B) A new mother breastfeeds her infant. (C) A corporate executive takes a vacation after working extremely hard the past month. (D) A spoiled child acts out by throwing his toys at the wall when he doesn't get the Christmas gift he wanted. (E) A mother and father fight about whether their son should have the privilege of staying out late to attend a party.

368. A defense mechanism is best defined by Freud as: (A) A systematic process used to avoid confrontation (B) A thought process that operates at an unconscious level to help an individual reduce anxiety (C) Th e creation of acceptable excuses for unacceptable behavior (D) Th e transfer of feelings from the unconscious to the conscious (E) A thought process by which forbidden desires are acknowledged

369. Todd has had a crush on Donna for the past year, but he does not have the courage to ask her out. He is frustrated with himself and begins taking a kickboxing class at his local gym. Th is scenario best illustrates which defense mechanism? (A) Displacement (B) Projection (C) Reaction formation (D) Rationalization (E) Sublimation

370. Which of the following statements best illustrates rationalization? (A) Jay fails his math class and blames it on his teacher not liking him. (B) After fighting with her best friend, Annie starts an argument with her mother. (C) Janie feels so guilty about cheating, she confesses to her teacher. (D) Conner is a heavy smoker but disregards all the evidence that says smoking can kill you. (E) Jarred doesn't want to believe that his pastor could have molested his younger brother.

371. Tom is still in love with his girlfriend, who broke up with him last week, but he acts as if he doesn't care and is actually happy to be rid of her. Th is is an example of which of the following defense mechanisms? (A) Regression (B) Projection (C) Sublimation (D) Displacement (E) Reaction formation

372. According to Freud, what is the preconscious? (A) Another name for conscious (B) Th e opposing force for the unconscious (C) Th e part of the mind that is right below the conscious surface (D) Th e part of the mind that works directly with the id (E) Th e part of the unconscious that does not hold repressed desires

373. According to Freud, in what stage of psychosexual development does the Oedipus complex take place? (A) Oral (B) Anal (C) Latency (D) Phallic (E) Genital

374. The female version of the Oedipus complex is called (A) Victoria complex (B) Isabella complex (C) Pleasure complex (D) Electra complex (E) Octavia complex

375. According to Freud's psychosexual theory of development, a man's repression of sexual urges is a result of which of the following? (A) Fixation in the latency stage (B) Fixation in the oral stage (C) Fixation in the anal stage (D) Fixation in the genital stage (E) Fixation in the phallic stage

376. One major criticism of Freudian psychoanalytic theory is that it: (A) Focuses too much attention on sexual conflicts and fixations (B) Assumes all behaviors are learned during childhood (C) Is too pessimistic about the future of humanity (D) Focuses too much attention on the id and not enough on the ego (E) Gives too much power to conscious behavior

377. A three-year-old boy is rejecting his father and only wants to be around his mother. Freud would theorize the child is going through which phase? (A) Electra complex (B) Pleasure principle (C) Oedipus complex (D) Reality principle (E) Latency period

378. Grace realizes she got back an extra hundred dollars from the bank teller. She has to decide whether or not she should return to the bank and inform the bank teller of the mistake. Grace is currently in conflict between her: (A) Conscious and unconscious (B) Id and superego (C) Ego and superego (D) Preconscious and unconscious (E) Id and conscious

379. A fixation in the oral stage will include all of the following behaviors except: (A) Overeating (B) Low self-esteem (C) Sarcasm (D) Self-consciousness (E) Aggressiveness

380. James has been divorced twice. Now anytime he even goes out on a date with women, they tell him he is very misogynistic. James could be fixated in what psychosexual stage of development? (A) Oral (B) Anal (C) Phallic (D) Latency (E) Genital

381. The "anima," "animus," "persona," and "shadow" are all: (A) Archetypes in the collective unconscious according to Carl Jung (B) Parts of the drive for superiority according to Alfred Adler (C) Components of Karen Horney's beliefs on neurotic needs (D) Terms used by Sigmund Freud to explain the Oedipus complex (E) Roles encouraged by neo-Freudians

382. Which theory of personality emphasizes the value and importance of unconditional positive regard with regard to relationships? (A) Psychoanalytic psychology (B) Humanistic psychology (C) Cognitive psychology (D) Developmental psychology (E) Behavioral psychology

383. The "Big Five" personality characteristics are: (A) Emotionality, extroversion, openness, neuroticism, and agreeableness (B) Anxiety, extroversion, agreeableness, neuroticism, and sociability (C) Outgoing, conscientiousness, extroversion, agreeableness, and neuroticism (D) Openness, conscientiousness, extroversion, agreeableness, and neuroticism (E) Extroversion, neuroticism, anxiety, agreeableness, and openness

384. Research shows that individuals with a type A personality are more prone to: (A) Extroversion (B) Cardiac health problems (C) Poverty (D) Sexual dysfunction (E) Psychoticism

385. Which of the following statements is a good example of a Jungian archetype? (A) Owen, who is 37, still wants to please his domineering mother. (B) Erica does not want anyone to know she uses food stamps. (C) George runs for class president because he wants his classmates to believe he is a confi dent person. (D) Tanya consciously strives to become the best golf player on her team. (E) Joan, who is haunted by her memories of child abuse, seeks help by going to a psychotherapist.

386. Which of the following tests is an example of a projective test, consisting of a set of ambiguous pictures about which people are asked to tell a story? (A) MMPI-2 (B) Rorschach (C) LSAT (D) TAT (E) ASW

387. Which of the following terms does not describe the assumption behind Carl Rogers's self theory? (A) Unconditional positive regard (B) Congruency (C) Self-actualization (D) Empathic understanding (E) Extraversion

388. According to Carl Rogers, a client's personality is determined by measuring the difference between: (A) Introversion and extraversion (B) Ideal self and real self (C) Self-efficacy and self-esteem (D) Persona and shadow (E) Self-actualization and esteem needs

389. According to Albert Bandura, self-efficacy is best described as: (A) Th e way in which an individual views his or her self-worth (B) A voluntary decision to postpone a personal reward until a specific task is completed (C) An individual's personal beliefs regarding how capable he or she is in controlling events and completing tasks (D) An individual's social, political, and cultural views on issues that influence his or her learning potential (E) An individual's beliefs about how much control he or she has over choices he or she has and decisions he or she makes

390. Ted believes that when he graduates depends primarily on his motivation and determination. Th is thought process is called: (A) Self-efficacy (B) Self-actualization (C) Social cognition (D) Internal locus of control (E) External locus of control

391. Raymond Cattell claimed that 35 basic traits could describe all differences among personalities. He called these traits. (A) External (B) Internal (C) Social (D) Source (E) Diverse

392. Th e trait theory can best be defined as: (A) Th e analysis of how much personality or behavioral traits are influenced by genetics (B) Th e analysis of the structure of personality by classifying similarities and differences in personality characteristics (C) A factor analysis that studies common personality characteristics (D) Th e organization of personality traits using specifi c categories to describe all characteristics (E) Grouping individual behaviors based on interactions between particular personality characteristics

393. A true-false self-report questionnaire that describes a wide range of normal and abnormal behaviors is called: (A) Th ematic Apperception Test (B) Validity Test (C) Rorschach Test (D) Objective Personality Test (E) Minnesota Multiphasic Personality Inventory

394. Complete the following statement: Th e theory minimized the role of the unconscious. (A) Humanistic (B) Trait (C) Psychoanalytic (D) Behaviorist (E) Functionalist

395. According to Carl Jung, the collective unconscious consists of: (A) Inherent tendencies to help people develop their true potential (B) Mental processes of which we are unaware but which automatically influence our thought patterns (C) Ancient memories and symbols that are passed down from birth and shared by all people in all cultures (D) Forces that influence our behavior (E) Biological drives shared by all people in all cultures

396. Alfred Adler proposed that humans are motivated by: (A) Conscious drives (B) Neurotic needs (C) Empathic understanding (D) Social urges (E) Intrinsic motivation

397. Which of the following neo-Freudians believed that the major influence on personality development is found in the child-parent social interaction? (A) Adler (B) Horney (C) Jung (D) Rogers (E) Bandura 398. According to Alfred Adler, fictional finalism is best defined as: (A) Th e belief that people live by many ideals that have no relation to reality (B) Th e desire people have to do good for their community (C) An individual's need to be in complete control over his or her life (D) Th e desire for power that all human beings innately struggle with (E) Th e social urges all people are motivated by and the unique way individuals deal with those urges

399. Neo-Freudians agree with Freud on all of the following basic ideas except: (A) Importance of the unconscious (B) The division of the mind (C) The use of defense mechanisms (D) The importance of sexual drives and conflicts (E) The protection of the ego

400. Th e importance of our capacity for personal growth, development of our potential, and freedom to choose our destiny is the emphasis of which psychological theory? (A) Psychoanalytic psychology (B) Existentialism (C) Behaviorism (D) Humanism (E) Cognitive psychology

401. Stress is best defined as: (A) A subjective evaluation of a situation that we believe to be overwhelming (B) A threatening feeling that comes when we interpret a situation as more than our psychological or physiological resources can handle (C) A potentially harmful situation from which we can potentially sustain some harm or damage (D) A situation that we see as a challenge to our psyche (E) A measure of how much we can handle a potentially threatening situation

402. Lamar was asked to give blood. He has a terrible fear of doing so. He automatically thinks this will have negative effects on his well-being. Th is is an example of what type of appraisal? (A) Harm/loss (B) Challenge (C) Threat (D) Stress (E) Negative

403. Which of the following statements is true regarding the fight-flight response? (A) It can be triggered by physical stimuli that threaten our survival. (B) It directs a great source of energy from the brain to the muscles. (C) It calms the body down after the response to a stress stimuli has occurred. (D) It stimulates the thyroid gland to release a stress hormone called adrenaline. (E) It automatically reduces physiological stress triggers by slowing down the heart rate.

404. Physical symptoms such as headaches, muscle pain, and stomach problems brought on by psychological factors like worry and tension are called: (A) Resistance symptoms (B) Prolonged stress symptoms (C) Psychological symptoms (D) Psychosomatic symptoms (E) Appraisal symptoms

405. Stress appraisal stimulates which part of the brain? (A) Thalamus (B) Hypothalamus (C) Amygdala (D) Cerebrum (E) Medulla

406. When the adrenal medulla is activated by the sympathetic nervous system, is secreted. (A) Epinephrine (B) Dopamine (C) Serotonin (D) Acetylcholine (E) Glycogen

407. What are the three stages of the general adaptation syndrome (GAS)? (A) Alarm, fight, relaxation (B) Alarm, control, exhaustion (C) Resistance, alarm, homeostasis (D) Alarm, resistance, exhaustion (E) Resistance, exhaustion, relaxation

408. Which of the following examples best illustrates frustration? (A) A basketball coach loses his temper when his team loses a game they should have won. (B) Two wolves fight to become the leader of the pack. (C) A child starts crying when his mother says good-bye to him in preschool. (D) A spider eats a fl y. (E) A farmer kills a chicken to eat for dinner.

409. When we balance the demands of a potentially stressful situation with our ability to meet these demands, it is called: (A) Secondary appraisal (B) Threat appraisal (C) Harm/loss appraisal (D) Challenge appraisal (E) Primary appraisal

410. Eva's professor keeps telling her how the tests and quizzes she takes in his class are opportunities to demonstrate her understanding of the material. Eva's professor is attempting to elicit what kind of appraisal? (A) Harm/loss (B) Threat (C) Challenge (D) Primary (E) Secondary

411. What effect do harm/loss appraisals have that challenge appraisals do not have? (A) Lower physiological arousal (B) Higher levels of negative emotions (C) More psychological stimulation (D) A triggering of physiological arousal (E) Increased activity of the parasympathetic nervous system

412. In what stage of the general adaptation syndrome is there a breakdown to internal organs and a weakening of the immune system? (A) Alarm (B) Resistance (C) Exhaustion (D) Relaxation (E) Negative

413. Just before her solo at her chorus concert, Charlene's heart begins to race and her face becomes flushed. According to Hans Selye, Charlene is in what stage of stress? (A) Alarm (B) Resistance (C) Exhaustion (D) Primary (E) Psychosomatic

414. Which of the following is not an example of a major source of stress? (A) Hassles (B) Change (C) Pressure (D) Frustration (E) Fear

415. Richard Lazarus's theory on stress emphasizes which of the following as the first step in experiencing stress? (A) Fear (B) Threat (C) Flight (D) Appraisal (E) Threat

416. A mental disorder is generally defined as: (A) Not knowing the difference between right and wrong (B) A prolonged problem that interferes with an individual's ability to cope in society (C) An anxiety disorder with dangers of hurting oneself (D) A long-term problem that can only be cured with medication (E) A long-term problem that cannot be treated with medication

417. Lee is unable to tell the difference between right and wrong in any aspect of his life. Th is statement is describing which type of abnormal behavior? (A) Depression (B) Maladaptive (C) Insanity (D) Anxiety (E) Psychotic

418. Th e learning perspective states that the main cause of mental disorders is: (A) Reinforcement of maladaptive behavior learned through experience (B) Irrational thought processes (C) Internal conflict from one's childhood (D) Low self-esteem (E) Chemical imbalance in the brain

419. DSM-IV was designed to help with which of the following? (A) Identifying psychological disorders (B) Identifying the causes of psychological disorders (C) Classifying psychological disorders (D) Listing venues where individuals can diagnose their disorder (E) Distinguishing between sanity and insanity

420. A somatoform disorder can best be defined as: (A) Disorder in which hallucinations occur often (B) Disorder in which an individual experiences extreme anxiety (C) Disorder in which symptoms are completely made up by the individual (D) Disorder in which symptoms are produced by psychological factors (E) Disorder in which an individual has delusional thoughts

421. Which of the following examples best illustrates a person with obsessive compulsive disorder (OCD)? (A) Steven hyperventilates whenever he is in an elevator. (B) Shelly complains constantly about feeling sick and goes to many doctors. (C) Bari is extremely anxious and panics every time she gets on an airplane. (D) Blake wanders around town in a daze, not sure how she got there. (E) Adam must lock his door 10 times before he leaves for work every morning.

422. A soldier experiences sudden blindness after returning from battle. He would most likely be diagnosed with which of the following disorders? (A) Conversion disorder (B) Dissociative disorder (C) Bipolar disorder (D) Hypochondriac (E) A phobic disorder

423. Which of the following disorders is not an anxiety disorder? (A) Phobias (B) Panic (C) Hypochondriasis (D) Obsessive-compulsive (E) Post-traumatic stress

424. Th is disorder is characterized by irritability, difficulty concentrating, and inability to control one's worry. (A) Phobias (B) Generalized anxiety (C) Obsessive-compulsive (D) Bipolar (E) Hypochondriasis

425. Fran was sitting on the bus when she suddenly felt overwhelmed. Her heart started racing, her legs began to feel weak, and her body trembled. She thought she was losing her mind. Fran's symptoms indicate she has: (A) Bipolar disorder (B) Panic disorder (C) Schizophrenia (D) Obsessive-compulsive disorder (E) Personality disorder

426. Agoraphobia is the fear of: (A) Heights (B) Spiders (C) Th e dark (D) Being in places with no escape (E) Speaking in public

427. Which of the following symptoms is not a symptom of obsessive compulsive disorder? (A) Irrational thoughts (B) Impulsive behavior (C) Uncontrollable images (D) Severe depression (E) Ritualized behavior

428. Individuals who have reported paralysis of a limb, blindness, or seizures with no physical or neurological damage are most likely suffering from: (A) A conversion disorder (B) A panic disorder (C) Post-traumatic stress disorder (D) Hypochondriasis (E) Bipolar disorder

429. Axis II of the DSM-IV refers to which of the following? (A) Mood disorders (B) Personality disorders (C) Anxiety disorders (D) Schizophrenia (E) General medical conditions

430. Which of the following disorders has psychological stressors translating into physical symptoms? (A) Anxiety (B) Adjustment (C) Affective (D) Somatoform (E) Psychotic

431. Data suggests that the most common mental disorder is: (A) Substance abuse (B) Mood disorders (C) Personality disorders (D) Somatoform disorders (E) Psychosexual disorders

432. Which of the following treatments is most often used to help clients who suffer from obsessive-compulsive disorder? (A) Avoidance therapy (B) Psychoanalysis (C) Exposure therapy (D) Biochemical treatment (E) Cognitive therapy

433. A list of criteria and symptoms about the onset, severity, and duration of mental disorders is located in which axis of the DSM-IV? (A) Axis I (B) Axis II (C) Axis III (D) Axis IV (E) Axis V

434. Th e theory that states that mental disorders develop when a biological predisposition to the disorder is set off by stressful circumstances is: (A) Cognitive-behavioral model (B) Psychoanalytic model (C) Diathesis-stress model (D) Biochemical model (E) Developmental model

435. Gender-identity disorders involve: (A) Th e use of unconventional sexual tendencies (B) Th e desire to dress like individuals of the opposite sex (C) Homosexual tendencies (D) Th e rejection of one's biological gender (E) Th e rejection of gender-related stereotypes

436. Which of the following disorders does not fall under a mood disorder? (A) Dysthymic disorder (B) Bipolar disorder (C) Major depression (D) Cyclothymic disorder (E) Schizophrenia

437. Lilly is now 35 years old. She just started therapy because she feels "down in the dumps." While in therapy she realizes she has felt this way most of her life. She is most likely suffering from: (A) Major depression (B) Bipolar disorder (C) Dysthymic disorder (D) Generalized anxiety disorder (E) Antisocial personality disorder

438. Antidepressant drugs work mainly because they raise the level of a single neurotransmitter called: (A) Dopamine (B) Epinephrine (C) Norepinephrine (D) Serotonin (E) Glycogen

439. Which of the following characteristics in not a symptom of a personality disorder? (A) Major depression (B) Inflexibility (C) Maladaptive traits (D) Impaired functioning (E) Great social and personal distress

440. Jeff has total disregard for the rights or properties of others. He steals all the time from just about anyone. He randomly harasses people. He has consistently destroyed his neighbor's property. Last month he was arrested for kidnapping. While in jail he continues to lie and have little remorse for his actions. Jeff is suffering from what mental disorder? (A) Major depression (B) Psychopath (C) Schizoid personality disorder (D) Dependent personality disorder (E) Paranoid personality disorder

441. Which of the following symptoms best illustrates schizoid personality disorder? (A) Disregard for the rights of others, feeling little to no remorse for bad behavior (B) Submissive behavior, excessive need to be taken care of (C) Acute discomfort in close relationships, distorted thinking, and eccentric behavior (D) Intense desire to be orderly, having total control over others (E) Excessively emotional and delusional, accompanied by a strong need for attention

442. Which of the following characterizes paranoid personality disorder? (A) Unstable moods (B) Lack of social relationships (C) Lack of conscience (D) Inaccurate sense of self-worth (E) Extreme suspiciousness and mistrust of other people

443. Early childhood sexual or physical abuse is a common feature among people suffering from: (A) Somatoform disorder (B) Dissociative identity disorder (C) Bipolar disorder (D) Major depression (E) Schizophrenia

444. From the time he was a young child, Scott has had no problem lying to authority figures. As an adult he considers himself good with the ladies. He has little remorse for his maladaptive behavior. Scott would most likely be diagnosed with: (A) Antisocial personality disorder (B) Paranoid personality disorder (C) Narcissistic personality disorder (D) Schizoid personality disorder (E) Schizophrenia

445. Pricilla spent the last four weeks in bed. Without telling her friends or family she bought a three-thousand-dollar plane ticket to Europe. She took most of her savings with her to go on a major shopping spree when she gets there. Pricilla is most likely suffering from: (A) Narcissistic personality disorder (B) Major depression (C) Schizoid personality disorder (D) Bipolar disorder (E) Dysthymic disorder

446. Excessive dopamine is to as too little dopamine is to . (A) Parkinson's disease, schizophrenia (B) Schizophrenia, Parkinson's disease (C) Antisocial personality disorder, schizoid personality disorder (D) Depression, schizophrenia (E) Schizophrenia, depression

447. All of the following are symptoms of schizophrenia except: (A) Delusions (B) Hallucinations (C) Disorganized speech (D) Manic behavior (E) Decreased emotional expression

448. Robert has been immobile for the past two years. In fact, he keeps both his arms up in the air for two-hour periods throughout the day. Robert has been diagnosed with: (A) Somatoform disorder (B) Paranoid schizophrenia (C) Conversion disorder (D) Disorganized schizophrenia (E) Catatonic schizophrenia

449. Which of the following is an example of a positive symptom of schizophrenia? (A) Hallucinations (B) Dulled emotions (C) Little inclination to speak (D) Loss of normal functions (E) Intellectual impairment

450. Research has shown that individuals with schizophrenia reportedly have a: (A) Larger hypothalamus (B) Smaller hypothalamus (C) Larger thalamus (D) Smaller thalamus (E) Smaller medulla

451. Khloe walked into a police station looking disheveled and confused. She could not remember her name, didn't recall where she came from, and couldn't remember anything about her past. Khloe has experienced: (A) Dissociative amnesia (B) Dissociative fugue (C) Dissociative identity disorder (D) Schizophrenia (E) Antisocial personality disorder

452. Researchers have determined that there is a genetic marker in the development of schizophrenia. To test this theory, researchers used which of the following groups? (A) Fraternal twins (B) Siblings (C) Parents and children (D) Unrelated individuals (E) Identical twins

453. Research suggests there is a direct correlation between the presence of major depression and: (A) Moderate levels of dopamine (B) Decreased levels of serotonin (C) Increased levels of endorphins (D) Enlarged hypothalamus (E) Enlarged parietal lobe

454. Autism is considered to be a: (A) Developmental disorder (B) Mood disorder (C) Learning disability (D) Personality disorder (E) Dissociative disorder 455. Narcissistic personality disorder is characterized by: (A) An unstable self-image (B) Feelings of inadequacy (C) Social isolation (D) Inflated sense of self (E) Compulsive tendencies 456. Th e analysis of a client's past experiences and suggestions for ways the client can overcome his or her problems that stem from these experiences is the basis for which type of therapy? (A) Cognitive therapy (B) Behavioral therapy (C) Psychoanalytic therapy (D) Developmental therapy (E) Social-cognitive therapy

457. Which of the following examples best illustrates insight therapy? (A) A client takes various psychoactive drugs to treat a mental disorder. (B) A therapist and client work together with the goal of identifying the problem and reaching a possible solution. (C) Th e therapist and client discuss key traumatic issues faced by the client in his or her childhood. (D) Th is therapy involves combining various techniques from many different therapeutic approaches. (E) Th e therapist focuses on the thoughts of the unconscious and brings these thoughts to the surface through dream analysis.

458. Which of the following is one major difference between a clinical psychologist and a psychiatrist? (A) A psychiatrist uses biomedical treatment. (B) A psychiatrist uses an eclectic approach. (C) A psychiatrist cannot counsel clients. (D) A psychiatrist recognizes the importance of group therapy. (E) A psychiatrist treats clients in hospitals.

459. Which of the following disorders has, in some circumstances, been treated with electroconvulsive therapy? (A) Paranoid personality disorder (B) Obsessive-compulsive disorder (C) Schizophrenia (D) Major depression (E) Dissociative identity disorder

460. Joy's therapist is trying to encourage her to take charge of the therapy session. The therapist uses active listening while Joy discusses her feelings. Which therapy is most likely being described? (A) Psychodynamic therapy (B) Rational emotive therapy (C) Existential therapy (D) Cognitive-behavioral therapy (E) Client-centered therapy

461. Th e antidepressant drug Prozac does which of the following? (A) Blocks the reuptake of serotonin (B) Blocks the reuptake of dopamine (C) Levels the amount of epinephrine (D) Deceases the amount of adrenaline in the blood stream (E) Decreases the level of acetylcholine in the blood stream

462. Albert Ellis devised a therapy that can be very confrontational. Th e client must face the irrationality of his or her belief system. What is the name of this form of therapy? (A) Cognitive-behavioral therapy (B) Gestalt therapy (C) Rational emotive therapy (D) Insight therapy (E) Social-cognitive therapy

463. Aversive therapy refers to: (A) An operant conditioning therapy that uses negative reinforcement to continued behavior (B) A classically conditioned therapy based on the theory that repeated pairings of negative effects lead to extinction (C) Using generalization to let all negative behavior pairings occur (D) Spontaneous recovery occurring long after a behavior that was based on negative pairing has ended (E) The use of modeling behavior so clients can see the consequences of negative behaviors

464. Gestalt therapy includes which of the following? (A) Free association (B) Electro-shock therapy (C) Behavioral therapy (D) Dream analysis (E) Biomedical therapy

465. The process by which a client expresses strong emotion toward the therapist is known as which of the following? (A) Transference (B) Free association (C) Dynamic therapy (D) Resistance (E) Projection

466. Which of the following is a major goal of Aaron Beck's cognitive therapy? (A) To rid an individual of his or her internal negative thought process (B) To change an individual's negative behavior (C) To enable a person to become self-actualized (D) To stop individuals from using selective attention (E) To help a client change learned or modeled behavior

467. Jana wants to be a doctor when she grows up, but she has one serious problem she is terribly afraid of blood. Since she was a little girl she has passed out at the mere sight of blood. To overcome this fear so that she can pursue her dream of becoming a doctor, her therapist exposes her to blood while trying to relax her. What type of therapy is this? (A) Social-cognitive therapy (B) Systematic desensitization (C) Behavioral therapy (D) Rational emotive therapy (E) Extinction

467. Jana wants to be a doctor when she grows up, but she has one serious problem; she is terribly afraid of blood. Since she was a little girl she has passed out at the mere sight of blood. To overcome this fear so that she can pursue her dream of becoming a doctor, her therapist exposes her to blood while trying to relax her. What type of therapy is this? (A) Social-cognitive therapy (B) Systematic desensitization (C) Behavioral therapy (D) Rational emotive therapy (E) Extinction

468. One major difference between a humanist therapist and a behavioral therapist is that: (A) A behavioral therapist focuses on one's childhood. (B) A humanist therapist pays attention to uncovering unconscious conflict. (C) A behavioral therapist can off er medication to her or his clients. (D) A humanist therapist focuses more on empathy and support for her or his clients. (E) A behavioral therapist places all of the burden on the client for her or his own healing.

469. Rational emotive therapy was designed to: (A) Teach clients relaxation techniques (B) Explore the unconscious confl icts from a client's childhood (C) Challenge the self-defeating thoughts of the client (D) Use antidepressant medication to overcome depression (E) Use free association to uncover unconscious thoughts and feelings

470. The purpose of free association is to: (A) Help bring unconscious confl ict to the surface (B) Facilitate changing negative behaviors (C) Change the client's thought process (D) Rid an individual of his or her sexual desires (E) Help a patient relax

471. Light therapy is used to help which of the following disorders? (A) Major depression (B) Dysthymic disorder (C) Obsessive-compulsive disorder (D) Dissociative identity disorder (E) Seasonal aff ective disorder

472. Which of the following terms is not associated with psychoanalysis? (A) Self-actualization (B) Free association (C) Dream analysis (D) Hypnosis (E) Sexual impulse

473. Which of the following psychologists believed that some people tend to have a pessimistic explanatory style, characterized by the tendency to blame bad events on themselves? (A) Aaron Beck (B) Martin Seligman (C) Karen Horney (D) Sigmund Freud (E) Abraham Maslow

474. What is the name of the widely used therapy that involves giving an individual immediate information about the degree to which he or she can change anxiety-related responses, thereby improving control over his or her physiological process of arousal? (A) Behavior modifi cation (B) Systematic desensitization (C) Behavioral therapy (D) Biofeedback (E) Cognitive therapy

475. Which of the following therapies has been found aff ective in treating anxiety disorders, drug addictions, and autism? (A) Psychoanalysis (B) Social-cognitive therapy (C) Behavioral therapy (D) Biomedical feedback (E) Gestalt therapy

476. John F. Kennedy's Bay of Pigs failure was caused in large part by: (A) Brainstorming (B) Group cohesion (C) Groupthink (D) Deindividuation (E) Diff usion of responsibility

477. Solomon Asch is most famous for his research on: (A) Conformity (B) Obedience (C) Compliance (D) Cohesion (E) Polarization

478. When we perform well on a task we typically attribute our success to our internal characteristics. Th is is known as: (A) Fundamental attribution error (B) Self-serving bias (C) Self schema (D) External attribution error (E) Person schema

479. Th e Stanford Prison experiment was a prime example of which of the following concepts? (A) Conformity (B) Compliance (C) Obedience (D) Cohesiveness (E) Identifi cation

480. According to the theory of cognitive dissonance, attitudes are changed because: (A) We are rewarded by society when our beliefs coincide with the majority. (B) Logical arguments compel us to alter our attitudes. (C) Emotionally persuasive arguments motivate us to change our thought process. (D) A state of tension motivates us to change our cognitive inconsistencies by making our beliefs more consistent. (E) When our beliefs and behaviors are too similar it causes an unpleasant psychological state of tension.

481. A person who agrees to a small request initially is more likely to comply with a larger demand later. Th is describes which phenomenon? (A) Door-in-face eff ect (B) Foot-in-door eff ect (C) Low-ball technique (D) High-ball technique (E) Door-in-foot technique

482. In Milgram's experiment, subjects who gave large shocks rationalized that they were not personally responsible for their actions. Th is raises questions about our willingness to commit inhumane acts as a result of: (A) Coercive power (B) Expert infl uence (C) Obedience to authority (D) Conformity to group pressure (E) Individual compliance

483. Which of the following was a factor in determining the degree of obedience in Milgram's series of experiments? (A) Distance between the teacher and the learner (B) Tone of voice of the teacher (C) Whether or not the teacher was male or female (D) Whether or not the teacher was an expert in his or her fi eld (E) Th e age of the teacher

484. In a situation in which an individual is having a seizure on the street, helping could be inhibited by which of the following concepts? (A) Groupthink (B) Social comparison theory (C) Risky shift (D) Diff usion of responsibility (E) Compliance

485. When making the "attribution error," we tend to overestimate the importance of when judging the behaviors of others. (A) Situational factors (B) Personal factors (C) Gender (D) Intelligence (E) Age

486. Th rough his experiments, Solomon Asch was able to demonstrate that: (A) People will always conform in a group setting. (B) Obedience to authority is determined by the perceived power of the authority figure. (C) Size of majority does not influence how many people will conform. (D) Compliance occurs in large groups. (E) Lack of unanimity greatly reduces the pressure to conform.

487. One reason why many groups have some form of initiation rites and rituals is to have: (A) Group norms (B) Deindividuation (C) Group cohesion (D) Task-oriented groups (E) Socially oriented groups

488. The Lapierre experiment proved that: (A) People's behavior usually corresponds with their attitudes. (B) People's attitudes do not necessarily reflect their behavior. (C) People tend to lie when asked to fi ll out a survey. (D) People are obedient in front of any person of authority. (E) Most people conform because of fear of embarrassment.

489. Damion rewrote his paper at the suggestion of his professor, even though he did not agree with the suggestions. Th is is an example of: (A) Obedience (B) Conformity (C) Compliance (D) Diff usion (E) Cognitive dissonance

490. Which of the following scenarios is an example of deindividuation? (A) Cindy fi nds that working in her group brings high levels of performance compared to students who work alone. (B) Mindy forms a study group because she wants academic help, social support, and motivation. (C) Amy has a poor running performance in competition; she performs even worse in front of a larger crowd. (D) Torrie honks her horn loudly for quite a while because she has little chance of being personally identified. (E) Jamie does not help the girl being attacked because the other bystanders are taking little action.

491. When group discussions change individuals' judgments, it is known as: (A) Risky shift (B) Groupthink (C) Group polarization (D) Social comparison (E) Group cohesion

492. Which of the following examples best illustrates a way to avoid groupthink from occurring? (A) Choose a group captain to make all the fi nal decisions. (B) Allow the group's members the freedom to express diff ering opinions. (C) Have every group member come in with a specifi c idea to bring to the table. (D) Only allow one person in the group to speak at a time. (E) Make the group socially oriented before making any fi nal decisions.

493. Of the following examples, which would be the best example of selfserving bias? (A) Michael, who believes that everyone should give to charities (B) Paris, who believes she failed her math test even though she always gets an A in math (C) Janet, who is always her teacher's favorite student (D) Randy, who believes he works harder than others and is underappreciated (E) Rebi, who overestimates her ability to run the after-school program for young children

494. After Jean was told by one of her professors that she would never succeed in law school, she stopped reading and completing her assignments. Eventually Jean did drop out of law school. Th is is an example of: (A) Self-fulfi lling prophecy (B) Self-serving bias (C) Social loafi ng (D) Groupthink (E) Diff usion of responsibility

495. David has always opposed the death penalty, believing it is not the place of the government to take the life of another person. After his best friend was murdered, David wanted nothing more than to see the murderer get the justice he or she deserved. Because the murder occurred in the state of Texas, this would mean justice would be served with the death penalty. Th e dissonance theory would state that: (A) David would have no confl ict in seeing the murderer put to death. (B) David would have to change one of his attitudes to feel less tension. (C) Justifi cation of the death penalty would be appropriate in this situation. (D) Morally, David would not support the death penalty under any circumstance. (E) David would change his opinion in support of the death penalty.

496. Th e tendency to attribute our own behavior to situational causes and the behavior of others to personal causes is an example of: (A) Self-fulfi lling prophecy (B) Actor-observer bias (C) Dispositional attribution (D) Attribution theory (E) Just-world phenomenon

497. Evidence suggests that individuals tend to be attracted to others who are: (A) Nearly opposite in all areas (B) Similar to themselves in terms of perspective and values (C) Physically more attractive than they are (D) Unlikely to criticize or judge them (E) Less intelligent than themselves

498. Th e tendency to "blame the victim" in a rape case is an example of which of the following terms? (A) Fundamental attribution error (B) Deindividuation (C) Self-serving bias (D) Th e just-world phenomenon (E) Self-fulfi lling prophecy

499. In the presence of the largest crowd she has ever seen, Heather gives her fi nest piano performance. Th is is an example of: (A) Group cohesion (B) Deindividuation (C) Group polarization (D) Social inhibition (E) Social facilitation

500. According to the diff usion of responsibility theory, the biggest factor in predicting whether or not a bystander will help someone in need is: (A) Th e duration of the situation (B) Whether or not the person in need of help is male or female (C) Th e number of other bystanders at the scene (D) Th e level of perceived threat (E) Whether or not the person actually asked for help

Which of the following situations best illustrates how a stimulus motive can direct behavior?

Which of the following situations best illustrates how a stimulus motive can direct behavior? children who see an object new to their environment will usually pick it up and try to see how it works.

Which of the following is an example of an avoidance avoidance conflict?

Avoidance-avoidance conflict is when a person has difficulty choosing between two unfavorable options. Examples of this include choosing between surgery or radiation treatments for cancer, or choosing between a lower salary at work or unemployment.

Which of the following theories suggests that a physiological need creates a state of tension?

Instinct theory was replaced by drive-reduction theory: the idea that a physiological need creates an aroused tension state (a drive) that motivates an organism to satisfy the need.

Which theory of emotion best describes that we adapt to ensure survival and safety?

Evolutionary Theory of Emotion Naturalist Charles Darwin proposed that emotions evolved because they were adaptive and allowed humans and animals to survive and reproduce. Feelings of love and affection lead people to seek mates and reproduce.